Está en la página 1de 40

EXÁMEN FINAL PSIQUIATRÍA 6TO SEMESTRE HPOL

1. Cuánto tiempo mínimo de síntomas es necesario para el diagnóstico de depresión ?


a. 1 mes
b. 2 semanas
c. 2 meses
d. 1 año
e. 5 días
2. Cuánto tiempo mínimo de síntomas es necesario para el diagnóstico de Trastorno de Ansiedad Generalizada ?
a. 1 mes
b. 2 semanas
c. 2 años
d. 6 meses
e. 1 año
3. Cuánto tiempo mínimo de síntomas es necesario para el diagnóstico de Distimia ?
a. 1 mes
b. 2 semanas
c. 2 años
d. 6 meses
e. 1 año
4. Cuánto tiempo mínimo de síntomas es necesario para el diagnóstico de Esquizofrenia ?
a. 2 mes
b. 2 semanas
c. 2 años
d. 6 meses
e. 1 año
5. Cuánto tiempo mínimo de síntomas es necesario para el diagnóstico de Trastorno Esquizofreniforme ?
a. 2 mes
b. 2 semanas
c. 1 mes
d. 6 meses
e. 1 año
6. Por cuánto tiempo se recomienda tratar farmacológicamente el primer episodio depresivo?
a. 1 mes
b. 2 meses
c. 1 año
d. 2 años
e. Hasta lograr la mejoría completa y suspender
7. Tratamiento farmacológico de primera línea para Trastorno de Pánico ?
a. Paroxetina 20mg 1 tab PO AM
b. Aripiprazol 15mg 1 tab PO HS
c. Diazepam 10mg IV al momento del ataque de pánico
d. Quetiapina 300mg 1 TAB PO BID
e. Cualquiera de los anteriores son adecuados
8. Tratamiento farmacológico recomendado de primera línea para TOC ?
a. Fluoxetina 20mg 1 TAB PO AM
b. Paroxetina CR 25 mg 2 TAB PO AM
c. Sertralina 50mg 1 TAB PO AM
d. Escitalopram 5mg 1 TAB PO AM
e. Cualquier ISRS en dosis bajas.
9. Cuál de estos no es un antipsicótico ?
a. Quetiapina
b. Risperidona
c. Olanzapina
d. Clozapina
e. Todos son antipsicóticos
10. Cuál son los dos principales componentes necesarios para diagnosticar Autismo?
a. Patrones Restrictivos/repetitivos en su comportamiento y Deficiencias en la interacción social.
b. Retraso Mental y Deficiencias en la interacción social.
c. Coeficiente Intelectual límite sin llegar al retraso más Deficiencias en la interacción social.
d. Irritabilidad ante estímulos sensoriales exagerados y Deficiencias en la interacción social.
e. Irritabilidad ante estímulos sensoriales exagerados y Retraso mental.
11. Cuáles son los episodios en que se puede presentar un Trastorno Bipolar?
a. Mixto, depresivo, eutímico, Maniaco
b. Maniaco o depresivos
c. Maniaco, hipomaniaco o depresivo
d. Maniaco, hipomaniaco, eutímico, depresivo, mixto
e. Maniaco, hipomaniaco, eutímico, depresivo, mixto, eufórico
12. Qué significa que un Trastorno bipolar tenga ciclación rápida ?
a. Que tenga más de 4 episodios anuales de recaídas depresivas o maniacales
b. Que tenga más de 6 episodios anuales de recaídas depresivas o maniacales
c. Que tenga más de 4 episodios mensuales de recaídas depresivas o maniacales
d. Que tenga más de 4 episodios semanales de recaídas depresivas o maniacales
e. Que tenga múltiples cambios de estado de ánimo incluso dentro de un mismo día que van de estar deprimido
a estar eufórico.
13. Indique el tiempo de tratamiento que se recomienda en el Trastorno Bipolar ?
a. 6 meses
b. 2 años
c. 1 año
d. De por vida
e. Hasta lograr la mejoría completa y suspender
14. Paciente de 15 años de edad, acude a consulta ambulatoria general, acompañada de sus padres, refieren que han
encontrado en su habitación una carta de despedida al parecer con objetivos suicidas, al entrevistar a la paciente ella
niega deseos suicidas sin embargo se muestra muy renuente y negativa durante la entrevista, cuál es su siguiente
paso como médico General en esta situación?
a. Explicar a los padres que probablemente su hija necesite atención Psicológica dado que podría estar
atravesando una depresión.
b. Iniciar tratamiento antidepresivo como Fluoxetina.
c. No dar ninguna explicación especial dado que esto es algo frecuente y la paciente esta manipulando la
situación.
d. Enviar a emergencia psiquiátricas a la paciente para probable internamiento.
e. Llamar a una nueva consulta en un mes y explicar a los padres que deben estar atentos de la paciente.
15. Cuál de estos tratamientos farmacológicos es específico para Demencia de Alzheimer.
a. Memantina + Fluoxetina
b. Rivastigmina + Memantina
c. Rivastigmina + Fluoxetina
d. Quetiapina + Fluoxetina
e. Risperdona + Memantina
16. Estos son tipos de demencia excepto ?
a. D. Fronto-vascular
b. D. de Cuerpos de Lewy
c. D. Alcohólica
d. D. en Párkinson
e. D. de Alzheimer
17. Estos son tipos de trastornos disociativos Excepto:
a. Amnesia Disociativa
b. Disociativa
c. Trastorno de Despersonalización.
d. Escape Disociativo
e. Trastorno de identidad Disociativo
18. Mujer de 26 años sin antecedentes psiquiátricos previos, acude a Urgencias traída por sus padres que explican que
desde hace unos días está hiperactiva, nerviosa, insomne. Explican que revisa obsesivamente la instalación
eléctrica de casa en busca de cámaras de vídeo y micrófonos. En la entrevista nos dice que le están vigilando desde
la Policía pues ella es una enviada galáctica con poderes especiales. ¿Cuál de las siguientes opciones NO debe
considerarse como diagnóstico diferencial?
a. Trastorno obsesivo compulsivo.
b. Esquizofrenia.
c. Episodio maníaco.
d. Psicosis por consumo de sustancias tóxicas.
e. Tumor cerebral.
19. Muchacha de 19 años estudiante, sin antecedentes personales somáticos ni psiquiátricos relevantes que es llevada
a urgencias hospitalarias por su familia por ingesta masiva de pastillas. La paciente fue encontrada por su madre
casualmente al volver a su casa antes de lo que tenía previsto. Una vez estabilizada orgánicamente la paciente
refería presentar desde unas 4 semanas atrás un cuadro de tristeza, anhedonia, desesperanza, sentimientos de
culpa y deseos de muerte, inhibición psicomotriz y enlentecimiento del pensamiento, dificultad de concentración y
rendimiento en los estudios, aislamiento social, hipersomnia diurna y empeoramiento matutino de los síntomas. Así
mismo la paciente refería tener la sensación de que sus vecinos la espiaban, hablaban de ella cuando salía y se
reían de ella lo que había incrementado su angustia. No sabía el motivo por el que la gente se fijaba en ella pero
estaba convencida de que no eran imaginaciones suyas. Señale el diagnóstico:
a. Esquizofrenia.
b. Trastorno de ideas delirantes persistentes.
c. Trastorno Psicótico breve
d. Trastorno distímico.
e. Trastorno Depresivo con síntomas psicóticos.
20. Una estudiante universitaria de 19 años acude a la consulta acompañada por sus padres refierendo sentirse en los
últimos dos meses progresivamente más asténica, con pérdida de apetito y de peso y con mayores dificultades para
concentrarse en los estudios. En la anamnesis también destaca que ha perdido interés en salir con las amigas,
presenta ideas de muerte sin ideación autolítica y cogniciones pesimistas de futuro. Su peso es el 90% del considerado
ideal por edad y género. No presenta fobia ponderal ni distorsión de la imagen corporal. El diagnóstico más adecuado
es:
a. Anorexia nerviosa.
b. Trastorno adaptativo mixto depresivo y ansioso
c. Trastorno adaptativo tipo depresivo.
d. Distimia.
e. Trastorno depresivo mayor
21. Mujer de 54 años, ex funcionaria pública de la salud, vive con pareja alcohólica. Diabética e hipertensa con
tratamientos irregulares en consultorios. No trabaja, vive de una pensión. Consume alcohol desde hace
aproximadamente 30 años, con múltiples tratamientos en servicios psiquiátricos, unidades especializadas, grupos
sociales. Llega a Servicio de Urgencia con nueva crisis de ingesta etílica, lo que ocurre con frecuencia, en estado de
ebriedad, llorosa y arrepentida, jurando que dejará de beber esta vez. Marcha atáxica y problemas con la pareja
estando bajo los efectos del alcohol. Manifiesta mucha angustia, sudor, taquicardia, polidipsia y polifagia. Trastornos
del Sueño. Dice que “no duerme hace dos días”. Bebiendo vino en forma intermitente. Cuál es la conducta a seguir
si recibes esta paciente en tu rural (Trabajas en Hospital Provincial básico):
a. Evaluar sobre conductas de abstinencia y tolerancia para transferir a Centro de especialidad en Addicciones
b. Atender de forma ambulatoria iniciar tratamiento y llamar a la paciente en un mes.
c. Evaluar glucosa en sangre.
d. Ingresar al Hospital básico para evaluación y tratamiento.
e. Transferir de emergencia a Hospital de alta complejidad de tercer nivel.
22. Mujer de 26 años de edad, que acude a la consulta de salud mental después de una hospitalización reciente por una
descompensación psicótica. Tras permanecer ingresada 22 días es dada de alta la semana previa a la revisión actual.
La familia refiere que la encuentran muy intranquila y con gran inquietud psicomotriz. En la exploración se evidencia
que no puede permanecer sentada y esta continuamente dando vueltas por la consulta sin un propósito concreto. No
se objetivan los síntomas psicóticos positivos que motivaron su ingreso, desde que en el hospital ajustaron su
tratamiento con 15 mg de haloperidol al día. El diagnóstico de la situación que presenta la paciente es:
a. Trastorno de Ansiedad Generalizada
b. Ansiedad secundaria a descompensación Psicótica
c. Acatisia
d. Trastorno por déficit de atención e Hiperactividad
e. Recaída Psicótica
23. ¿Cuál de las siguientes afirmaciones respecto a los trastornos de ansiedad es FALSA?
a. Pueden manifestarse como estados persistentes de inquietud psicomotora y nerviosismo.
b. La activación ansiosa puede alterar los niveles de cortisol y de otras hormonas.
c. Pueden manifestarse como estados bruscos y rápidos de inquietud psicomotora y nerviosismo.
d. La presencia de síntomas depresivos excluye el diagnóstico de trastornos de ansiedad
24. Mujer de 55 años, casada y con dos hijos de 25 y 20 años de edad. Acude a la consulta de salud mental por
problemas en el entorno familiar. La paciente refiere encontrarse mal, triste y preocupada por “los disgustos que me
da el marido”. Lo ha visto abrazado a otra mujer y afirma que “´él está en su contra y que la engaña porque se
preocupa demasiado por sus hijos”. Tanto el marido como los hijos desmienten esa información dada por la
paciente. Sin antecedentes médicos no psiquiátricos a destacar, presenta una exploración psicopatológica de
lucidez con buen nivel de conciencia y de orientación en las 3 esferas. Presenta hipotimia discreta de meses de
evolución. Alteración del contenido del pensamiento en forma de ideas delirantes de corte celotípico y de perjuicio
con fenómenos autorreferenciales. No existe alteración de la esfera sensoperceptiva. No presenta ideación
autolítica. Mantiene un adecuado ritmo cognitivo y conserva los ritmos biológicos de sueño y apetito. ¿Cuál es el
diagnóstico que se ajusta más al cuadro clínico que presenta la paciente?
a. Esquizofrenia paranoide.
b. Trastorno depresivo mayor con características psicóticas.
c. Trastorno paranoide la de personalidad
d. Trastorno de ideas delirantes persistentes.
e. Trastorno Depresivo con psicosis
25. ¿Cuál es la orientación diagnóstica ante un hombre de 35 años sin antecedentes médicos que acude a consulta
explicando temores insuperables a entrar en lugares concurridos tras haber experimentado hace un mes, al entrar
en un centro comercial, un episodio agudo de ansiedad con síntomas somáticos (taquicardia, sudores, disnea..)?
a. Trastorno por estrés anticipatorio.
b. Fobia de impulsación evitativa.
c. Trastorno obsesivo.
d. Trastorno por angustia con agorafobia.
e. Fobia Social.
1. Mujer de 54 años, ex funcionaria pública de la salud, vive con pareja alcohólica. Paciente es diabética e
hipertensa con tratamientos irregulares en consultorios. No trabaja, vive de una pensión de invalidez.
Consume alcohol desde hace aproximadamente 30 años, con múltiples tratamientos en servicios
psiquiátricos, unidades especializadas, grupos sociales. Llega al Servicio de Urgencia con nueva crisis
de ingesta etílica, lo que ocurre con frecuencia, en estado de ebriedad, llorosa y arrepentida, jurando
que dejará de beber esta vez. Marcha atáxica y problemas con la pareja estando bajo los efectos del
alcohol. Manifiesta mucha angustia, sudor, taquicardia, polidipsia y polifagia. Trastornos del Sueño.
Dice que “no duerme hace dos días”. Bebiendo vino en forma intermitente.

Cuál es la conducta a seguir si recibes esta paciente en tu rural (Trabajas en Hospital Provincial
básico):

1. Evaluar sobre conductas de abstinencia y tolerancia para transferir a Centro de especialidad en


Addicciones
2. Atender de forma ambulatoria iniciar tratamiento y llamar a la paciente en un mes.
3. Evaluar con exámenes de laboratorio de rutina y específicos por sus síntomas.
4. Ingresar al Hospital básico para evaluación y tratamiento.
5. Transferir de emergencia a Hospital de alta complejidad de tercer nivel.
2. Frente a un delirio celotípico hay que sospechar la existencia de:
1. SIDA.
2. Alcoholismo.
3. Demencia.
4. Personalidad esquizoide.
5. Depresión Grave
3. Señalar cuáles de los siguientes síntomas de la esquizofrenia constituye un factor de mal pronóstico:
1. Pródromo breve.
2. Ausencia de síntomas depresivos.
3. Inicio tardío.
4. Ausencia de síntomas obsesivos.
5. Presencia de Síntomas negativos
4. Señalar en que trastorno de la personalidad se observa la ausencia de remordimientos y la incapacidad
para planificar el futuro:
1. Trastorno paranoide de la personalidad.
2. Trastorno límite de la personalidad.
3. Trastorno antisocial de la personalidad.
4. Trastomo de la personalidad por evitación.
5. Trastorno Dependiente de la Personalidad.
5. Hombre de 26 años, soltero, que es traído a consulta por su familia por llevar 3 meses negándose a
salir se su casa. La razón que aduce el paciente es el convencimiento de que tiene la mandíbula
asimétrica y la cara torcida. Según refiere esta situación es progresiva y cada vez se ve mas deforme
cuando se observa en el espejo. Se avergüenza de su aspecto, por lo que no quiere salir, se angustia
mucho cuando ve su imagen y no puede dejar de pensar todo el día en su deformidad. Ha consultado
con varios cirujanos maxilofaciales pero éstos le dicen que no presenta asimetría facial y lo remiten al
psiquiatra. El diagnóstico del paciente es:
1. Trastorno depresivo mayor con ideas delirantes incongruentes con el estado de ánimo.
2. Trastorno obsesivo compulsivo.
3. Esquizofrenia paranoide.
4. Trastorno dismórfico corporal
5. Trastorno de Somatización.
6. Una paciente presenta una larga historia de múltiples síntomas físicos: pérdida de memoria, dolor de
cabeza, mareos, vómitos, dolor genital, dolor en extremidades, distensión abdominal e irregularidades
menstruales. Los diversos exámenes médicos han descartado enfermedad médica alguna. ¿Cuál de
los siguientes cuadros es más probable que padezca?
1. Trastorno conversivo.
2. Trastorno hipocondríaco.
3. Trastorno de somatización.
4. Trastomo disociativo.
5. Trastorno de Ideas delirantes Persistentes
7. Mujer de 17 años, estudiante, vive con sus padres y hermanos. Angustiada llora constantemente, decaída,
desanimada, ha dejado de salir a fiestas y ya no comparte con amigos. La noche anterior ingiere cantidad
no precisada de tabletas “para los nervios” que tomaba su madre. No es primera vez que lo hace, ésta es la
tercera vez y siempre ha sido luego de un problema. Refiere estar arrepentida por lo que hizo.

Cuál es el probable trastorno de personalidad que acompaña a esta mujer que acude con criterios para
depresión?

1. T. de personalidad Histriónica.
2. T. de personalidad Ciclitímica
3. T. de personalidad Obsesiva
4. T. de personalidad Esquizotípica
5. T. de personalidad Limítrofe
8. Paciente sexo masculino 30 años, casado. Acude con una carpeta con documentos bajo el brazo, buen
contacto, limpio y bien vestido. Relata que hace 6 meses que está con un cuadro depresivo severo, que
ha pasado por múltiples tratamientos y diferentes médicos y no ha mejorado. Con licencia médica
desde el inicio de su enfermedad, la cual se inició tras separación conyugal (por segunda vez) porque,
“ella no me comprendía”. Asegura ser un excelente trabajador y esposo. Niega consumo de drogas y/o
alcohol.Al tratar de establecer una sintomatología precisa del momento actual, aporta datos
contradictorios y poco claros. El aspecto, la presencia y el enfoque no están de acuerdo con lo que
relata (sin angustia, voz fuerte y firme, y sin compromiso emocional). Asegura enfáticamente que no
puede retomar su trabajo, razón por la que acude al Servicio de Urgencia, pidiendo un certificado dado
que el día siguiente tiene que presentarse a un proceso judicial por divorcio. Cuál es la impresión
diagnóstica más probable en este paciente?
1. T. de Personalidad.
2. T. disociativo de personalidad (Personalidad Múltiple)
3. Simulación
4. T. Disociativo posttraumático
5. T. Facticio.
9. Muchacha de 19 años estudiante, sin antecedentes personales somáticos ni psiquiátricos relevantes
que es llevada a urgencias hospitalarias por su familia por ingesta masiva de pastillas hace una
hora. Siendo la paciente encontrada por su madre casualmente al volver a su casa antes de lo que
tenía previsto. Paciente refiere presentar desde unas 4 semanas atrás un cuadro de tristeza,
anhedonia, desesperanza, sentimientos de culpa y deseos de muerte, inhibición psicomotriz y
enlentecimiento del pensamiento, dificultad de concentración y rendimiento en los estudios,
aislamiento social, hipersomnia diurna y empeoramiento matutino de los síntomas. Así mismo la
paciente refería tener la sensación de que sus vecinos la espiaban, hablaban de ella cuando salía y
se reían de ella lo que había incrementado su angustia. No sabía el motivo por el que la gente se
fijaba en ella pero estaba convencida de que no eran imaginaciones suyas. Señale el tratamiento
farmacológico en urgencias
1. Amitriptilina intravenosa.
2. Lavado gástrico y carbón activado.
3. Fluoxetina altas dosis por vía oral.
4. Diazepam intravenosa para sedar al paciente.
5. Haloperidol Intravenoso.
10. Mariano es un señor de 53 años de edad que acude a su consulta refiriendo sentir un gran malestar
desde hace ya unos años. Relaciona el malestar con una conducta que encuentra absurda pero que
es incapaz de evitar. Esta conducta que lleva a cabo siempre al llegar a su casa de vuelta del
trabajo consiste en accionar el pomo del baño del piso superior de la vivienda antes de hacer
cualquier otra cosa, incluso antes de saludar a su familia. Algunas veces ha intentado resistirse a
hacerlo pero solo ha conseguido angustiarse y demorar la conducta unos minutos. Éste
comportamiento que Mariano realiza de un modo incoercible y automático es lo que se denomina:
1. Comportamiento normal.
2. Obsesión.
3. Impulsión.
4. Compulsión.
5. Tic complejo
11. Tratamiento de primera línea para el trastorno obsesivo compulsivo.
1. Sertralina 100 a 200 mg PO AM diario
2. Paroxetina 60mg PO HS diaria.
3. Terapia Psicológica ambulatoria.
4. Todas son terapias de primera línea.
5. Ninguna es terapia de primera línea.
12. Profesor varón, de 40 años de edad, natural y procedente del Chota, quién desde hace 10 años
aproximadamente y de forma reiterada insiste en un reclamo a una familia que se apellida Belaunde
sobre una herencia que supuestamente le corresponde refiere en su reclamo judicial “Mis apellidos son
Vela Terry, pero para confundir a la opinión pública y a mí persona, el apellido del ex -presidente lo han
escrito con “B” y le han añadido el “unde por lo que las herencias del presidente de la República me
corresponderían”. ¿Señale Ud. cuál es su hipótesis diagnóstica más probable? Escoja una opción
1. Demencia de Alzheimer.
2. Trastorno orgánico Cerebral.
3. Delirium Crónico.
4. Trastorno de ideas delirantes persistentes.
5. Demencia mixta vascular y senil.
13. Caso clínico. Un estudiante varón de 20 años de edad, previamente sano, se ha estado comportando
de forma cada vez más extraña. A veces parece enfadado, comenta a sus amigos que está siendo
seguido por la policia y por servicios secretos, a veces ríe solo sin motivo. Últimamente se preocupa
más por sus pensamientos y su rendimiento académico ha bajado. Cuando fue a consulta estaba
inquieto y asustado. Decía que escuchaba voces que comentaban sus acciones. Además piensa que le
están robando sus pensamientos y que en la televisión hacen referencia a él, estos síntomas inician
según sus padres hace dos meses y han ido empeorando. Estos son parte de síntomas del criterio A
para esquizofrenia según el DSM – IV – TR, excepto
1. Disfunción social y ambiental
2. Alucinaciones
3. Delirios
4. Comportamiento desorganizado o catatónico
5. Síntomas negativos como aplanamiento afectivo.
14. Una mujer de 23 años, sin historia de abuso de alcohol, refiere episodios matinales de temblor,
ansiedad, taquicardia y sudoración. Al realizar el examen físico se evidencia dermografía negativa y
presencia de rinitis inespecífica.
¿Cuál de los siguientes exámenes tiene mayor probabilidad de estar alterado?

1. Amonemia
2. Glicemia
3. Potasio sérico
4. T3, T4, TSH
5. Pesquisa de drogas en orina

15. El tratamiento farmacológico completo de un paciente con un primer episodio de depresión mayor, sin
síntomas psicóticos, debe realizarse por un tiempo MINIMO de:

1) 2 semanas
2) 2 meses
3) 5 meses
4) 6 meses
5) 24 meses

16. Una estudiante universitaria de 19 años acude a la consulta acompañada por sus padres refierendo
sentirse en los últimos dos meses progresivamente más asténica, con pérdida de apetito y de peso
y con mayores dificultades para concentrarse en los estudios. En la anamnesis también destaca que
ha perdido interés en salir con las amigas, presenta ideas de muerte con ideación autolítica y
cogniciones pesimistas de futuro. Su peso es el 90% del considerado ideal por edad y género. No
presenta fobia ponderal ni distorsión de la imagen corporal. El diagnóstico más adecuado es:
1. Anorexia nerviosa.
2. Trastorno depresivo leve
3. Trastorno Depresivo grave con Psicosis
4. Distimia.
5. Trastorno depresivo grave.
17. Señale cual de los siguientes tratamientos NO está indicado para el tratamiento crónico del trastorno
Psicótico Breve.
1. Quetiapina
2. Olanzapina
3. Risperidona.
4. Metilfenidato.
5. Aripiprazol.
18. Señalar cual es el trastorno donde el paciente tiende a acudir al médico frecuentemente por el
simple hecho de buscar atención médica que incluso llega al hecho de enfermarse voluntariamente.
1. Trastorno de personalidad narcisista.
2. Trastornos disociativos.
3. Trastorno hipocondriaco.
4. Trastorno límite de la personalidad.
5. Trastorno facticio.
19. Qué fármaco podría causar un brote maniacal en un tratamiento de un paciente con depresión crónica
de 40 años de evolución y tratamiento antidepresivo contínuo en los últimos 30 años ?
1. Ninguno podría.
2. Bupropion (Wellbutrim)
3. Todos podrían.
4. Sertralina.
5. Venlafaxina.
20. Mujer de 60 años de edad que presenta ideación suicida, cortes profundos en muñecas, obsesiones
sobre contaminación, olvidos frecuentes, baja de peso, traída agitada por sus familiares, insulta en el
cuarto de emergencia y le dice a usted que por favor no le mate y se muestra entre agresiva y confusa,
cuál de estos diagnósticos es el menos probable que sea la causa de esta sintomatología?
1. TOC descompensado
2. Delirium.
3. Trastomo adaptativo depresivo.
4. Episodio de depresión mayor.
5. Demencia tipo Alzheimer.
21. Mujer de 26 años sin antecedentes psiquiátricos previos, acude a Urgencias traída por sus padres que
explican que desde hace unos días está hiperactiva, nerviosa, insomne. Explican que revisa
obsesivamente la instalación eléctrica de casa en busca de cámaras de vídeo y micrófonos. En la
entrevista nos dice que le están vigilando desde la Policía pues ella es una enviada galáctica con
poderes especiales. ¿Cuál de las siguientes opciones NO debe considerarse como diagnóstico
diferencial?
1. Trastorno obsesivo compulsivo.
2. Esquizofrenia.
3. Episodio maníaco.
4. Psicosis por consumo de sustancias tóxicas.
5. Tumor cerebral.
22. Referido a las formas clínicas de la esquizofrenia ¿cuál de las siguientes afirmaciones es cierta?
1. La forma CATATÓNICA se presenta con sintomatología pobre, predominio de delirios y suele
ser de inicio temprano.
2. En la forma HEBERFRÉNICA existe predominio de ateraciones psicomotrices, negativismo y
conductas esteriotipadas.
3. La presencia de estados de exaltación anímica, maniforme con extravagancias es propia de las
formas SIMPLES.
4. La forma PARANOIDE presenta predominantemente trastornos del pensamiento y trastornos
sensoperceptivos.
5. Los síntomas obsesivos y compulsivos asociados a disgregación progresiva son habitualmente
en la forma, ESQUIZOTÍPICA.
23. Cuál de las siguientes afirmaciones NO ES CIERTA para el trastorno bipolar?
1. Se presenta con la sucesión de fases depresivas y maníacas, aunque pueden presentarse
intervalos de normalidad.
2. La TEC (terapia electroconvulsiva) es indicación de primera linea en el trastorno bipolar durante
el embarazo.
3. La herencia admitida en el trastorno bipolar es de tipo autosómico recesiva con penetrancia
incompleta.
4. El tratamiento del trastorno bipolar con carbonato de litio está indicado tanto en fase maníaca
como para una terapéutica profiláctica.
5. La presencia de delirios organizados excluye el diagnóstico de trastorno bipolar.

24. La presencia de síntomas neurológicos sin explicación médica se explicaría mejor por cuál de las
siguientes enfermedades psiquiátricas?
1. Trastorno facticio.
2. Simulación.
3. Trastorno por conversión.
4. Trastorno somatomorfo
5. Todas son correctas
25. Los tres componentes básicos para el diagnóstico de autismo son:
1. Mala Interacción social, retraso mental y falta de juego espontáneo.
2. Imposibilidad de comunicar sentimientos, retraso mental y retraso en el lenguaje.
3. Conductas restrictivas, pobre interacción social y retraso o ausencia en el desarrollo del
lenguaje.
4. Déficit cognitivo, pobre interacción social y retraso en el lenguaje.
5. Afectación social/laboral; retraso en el lenguaje y que no sea causado por enfermedad médica.
26. Qué hipótesis neurobiológica no se relaciona con la depresión de una forma relevante:
1. Hipótesis hormonal relacionada con control de la FSH y la LH
2. Hipótesis genética relacionada con alteraciones génicas en la síntesis del recaptador de
serotonina.
3. Hipótesis de Estrés relacionada con alteración en la plasticidad cerebral como alteraciones en el
CREB y otro tipo de quinasa y activadores de secuencias repetitivas.
4. Hipótesis de alteración dopaminérgica.
5. Hipótesis aminérgica.
27. Mujer de 47 años sin antecedentes psiquiátricos que es hospitalizada para estudio de metrorragias y a
los 5 días de su ingreso es informada del diagnóstico de neoplasia uterina con metástasis. Se consulta
a Psiquiatría porque 24 horas después muestra tristeza y llanto frecuente, refiere ideas de muerte y
presenta insomnio. En la evaluación psiquiátrica no se recoge ningún antecedente psiquiátrico y la
exploración detecta elevada ansiedad y desesperanza en relación con las consecuencias de su
enfermedad neoplásica. El diagnóstico más probable es:

1. Trastorno esquizoafectivo.
2. Distimia.
3. Trastorno de ansiedad generalizada.
4. Trastorno por estrés postraumático.
5. Trastorno adaptativo.
28. Un señor de 60 años, empleado de comercio, acude a urgencias acompañado de su familia; éstos
cuentan que el día anterior, de forma bastante brusca, comenzó a decir cosas raras, a no responder a
lo que le preguntaban y a mostrar se confuso incluso respecto de su propio nombre. Efectivamente en
la exploración parece no entender lo que se le pregunta, no recuerda nada de lo que le ha pasado y no
sabe ni el día, ni el lugar en el que está. El diagnóstico sería:

1. Psicosis breve.
2. Delirium.
3. Alzheimer.
4. Esquizofrenia.
5. Trastorno de ansiedad
29. ¿En cuál de los siguientes casos en una mujer embarazada la terapia electroconvulsivante NO es un
tratamiento de primera elección?:

1. Esquizofrenia paranoide.
2. Psicosis reactiva.
3. Depresión mayor con ideación suicida.
4. Trastorno esquizotípico de la personalidad.
5. Trastorno de ideas delirante persistentes crónico.

30. Cuál de estos fármacos no es un ISRS?


1. Venlafaxina
2. Sertralina
3. Fluoxetina
4. Paroxetina
5. Escitalopram
31. Cuánto tiempo cómo máximo debe durar la sintomatología de un Trastorno Esquizofreniforme?
1. 2 semanas
2. 1 mes
3. 3 meses
4. 6 meses
5. Es crónico “de por vida”
32. Cuál son los síntomas cardinales en un cuadro depresivo?
1. Hipotimia y anhedonia
2. Astenia y bajo estado de ánimo.
3. Ideación suicida y baja de peso,
4. Alteraciones en apetitio y sueño.
5. Baja autoestima y tristeza.
33. Cuál de estos fármacos no es un eutimizante?
1. Olanzapina
2. Venlafaxina
3. Quetiapina
4. Litio
5. Acd. Valpróico
34. Paciente de 15 años de edad, acude a consulta ambulatoria general, acompañada de sus padres,
refieren que han encontrado en su habitación una carta de despedida al parecer con objetivos suicidas,
al entrevistar a la paciente ella niega deseos suicidas sin embargo se muestra muy renuente y negativa
durante la entrevista, cuál es su siguiente paso como médico General en esta situación?
1. Explicar a los padres que probablemente su hija necesite atención Psicológica dado que podría
estar atravesando una depresión.
2. Iniciar tratamiento antidepresivo como Fluoxetina.
3. No dar ninguna explicación especial dado que esto es algo frecuente y la paciente esta
manipulando la situación.
4. Enviar a emergencia psiquiátricas a la paciente para probable internamiento.
5. Llamar a una nueva consulta en un mes y explicar a los padres que deben estar atentos de la
paciente.
35. Cuál de estos fármacos antidepresivos no presenta como efecto colateral disminución de la líbido en los
hombres?
1. Bupropion XL.
2. Venlafaxina.
3. Fluoxetina.
4. Sertralina.
5. Mirtazapina.
36. Que es el vaginismo?
1. Aumento de flora anormal en la mucosa vaginal.
2. Contracción involuntaria de las paredes musculares vaginales.
3. Infección vaginal gonocócica.
4. Resequedad de las paredes vaginales.
5. Infección vaginal indeterminada.
37. Que es el Voyeurismo?
1. Placer a través del frotamiento de órganos genitales con objetos o personas.
2. Placer con uso de correas en el acto sexual.
3. Placer en la sensación de traicionar a la pareja sentimental.
4. Placer al sentir dolor o mediante juegos de dominación durante el acto sexual.
5. Placer al ver a otras personas en situaciones eróticas.
38. Estos son criterios para trastorno de ansiedad Generalizada excepto:
1. Ansiedad y preocupación excesivas
2. Inquietud o impaciencia; fatigabilidad fácil.
3. Tensión muscular, irritabilidad
4. El Tiempo mínimo de todos los síntomas es de un mes.
5. La sintomatología no debe ser causada por drogas.
39. Cuál es el principal diagnóstico diferencial en una fuga disociativa ?
1. Convulsión.
2. Demencia de Alzheimer
3. Delirium
4. Intoxicación con drogas
5. Depresión Psicótica.
40. Qué es un heteroreceptor en la neurotransmisión ?
1. Es un receptor diferente al neurotransmisor que lo activa.
2. En un receptor capaz de activarse por diferentes Neurotransmisores.
3. Es un receptor que se sitúa en la membrana de una neurona de diferente clase en relación al
ligando que lo estimula.
4. Es un receptor atípico el cual presenta una heterogenicidad en relación a la activación de la
protenia G.
5. Es un receptor que presenta dos subunidades diferentes la una con la otra.
41. PREGUNTA BONUS: Cuál es el rango normal terapeútico del lítio en Sangre? (mEq/L)
1. Mujer de 54 años, ex funcionaria pública de la salud, vive con pareja alcohólica. Paciente es diabética e
hipertensa con tratamientos irregulares en consultorios. No trabaja, vive de una pensión de invalidez.
Consume alcohol desde hace aproximadamente 30 años, con múltiples tratamientos en servicios
psiquiátricos, unidades especializadas, grupos sociales. Llega al Servicio de Urgencia con nueva crisis de
ingesta etílica, lo que ocurre con frecuencia, en estado de ebriedad, llorosa y arrepentida, jurando que
dejará de beber esta vez. Marcha atáxica y problemas con la pareja estando bajo los efectos del alcohol.
Manifiesta mucha angustia, sudor, taquicardia, polidipsia y polifagia. Trastornos del Sueño. Dice que “no
duerme hace dos días”. Bebiendo vino en forma intermitente.

Cuál es la conducta a seguir si recibes esta paciente en tu rural (Trabajas en Hospital Provincial básico):

1. Evaluar sobre conductas de abstinencia y tolerancia para transferir a Centro de especialidad en


Addicciones
2. Atender de forma ambulatoria iniciar tratamiento y llamar a la paciente en un mes.
3. Evaluar con exámenes de laboratorio de rutina y específicos por sus síntomas.
4. Ingresar al Hospital básico para evaluación y tratamiento.
5. Transferir de emergencia a Hospital de alta complejidad de tercer nivel.
2. Frente a un delirio celotípico hay que sospechar la existencia de:
1. SIDA.
2. Alcoholismo.
3. Demencia.
4. Personalidad esquizoide.
5. Depresión Grave
3. Señalar cuáles de los siguientes síntomas de la esquizofrenia constituye un factor de mal pronóstico:
1. Pródromo breve.
2. Ausencia de síntomas depresivos.
3. Inicio tardío.
4. Ausencia de síntomas obsesivos.
5. Presencia de Síntomas negativos
4. Señalar en que trastorno de la personalidad se observa la ausencia de remordimientos y la incapacidad
para planificar el futuro:
1. Trastorno paranoide de la personalidad.
2. Trastorno límite de la personalidad.
3. Trastorno antisocial de la personalidad.
4. Trastomo de la personalidad por evitación.
5. Trastorno Dependiente de la Personalidad.
5. Hombre de 26 años, soltero, que es traído a consulta por su familia por llevar 3 meses negándose a salir
se su casa. La razón que aduce el paciente es el convencimiento de que tiene la mandíbula asimétrica y la
cara torcida. Según refiere esta situación es progresiva y cada vez se ve mas deforme cuando se observa
en el espejo. Se avergüenza de su aspecto, por lo que no quiere salir, se angustia mucho cuando ve su
imagen y no puede dejar de pensar todo el día en su deformidad. Ha consultado con varios cirujanos
maxilofaciales pero éstos le dicen que no presenta asimetría facial y lo remiten al psiquiatra. El diagnóstico
del paciente es:
1. Trastorno depresivo mayor con ideas delirantes incongruentes con el estado de ánimo.
2. Trastorno obsesivo compulsivo.
3. Esquizofrenia paranoide.
4. Trastorno dismórfico corporal
5. Trastorno de Somatización.
6. Una paciente presenta una larga historia de múltiples síntomas físicos: pérdida de memoria, dolor de
cabeza, mareos, vómitos, dolor genital, dolor en extremidades, distensión abdominal e irregularidades
menstruales. Los diversos exámenes médicos han descartado enfermedad médica alguna. ¿Cuál de los
siguientes cuadros es más probable que padezca?
1. Trastorno conversivo.
2. Trastorno hipocondríaco.
3. Trastorno de somatización.
4. Trastomo disociativo.
5. Trastorno de Ideas delirantes Persistentes
7. Mujer de 17 años, estudiante, vive con sus padres y hermanos. Angustiada llora constantemente, decaída,
desanimada, ha dejado de salir a fiestas y ya no comparte con amigos. La noche anterior ingiere cantidad no
precisada de tabletas “para los nervios” que tomaba su madre. No es primera vez que lo hace, ésta es la tercera
vez y siempre ha sido luego de un problema. Refiere estar arrepentida por lo que hizo.

Cuál es el probable trastorno de personalidad que acompaña a esta mujer que acude con criterios para
depresión?

1. T. de personalidad Histriónica.
2. T. de personalidad Ciclitímica
3. T. de personalidad Obsesiva
4. T. de personalidad Esquizotípica
5. T. de personalidad Limítrofe
8. Paciente sexo masculino 30 años, casado. Acude con una carpeta con documentos bajo el brazo, buen
contacto, limpio y bien vestido. Relata que hace 6 meses que está con un cuadro depresivo severo, que ha
pasado por múltiples tratamientos y diferentes médicos y no ha mejorado. Con licencia médica desde el
inicio de su enfermedad, la cual se inició tras separación conyugal (por segunda vez) porque, “ella no me
comprendía”. Asegura ser un excelente trabajador y esposo. Niega consumo de drogas y/o alcohol.Al tratar
de establecer una sintomatología precisa del momento actual, aporta datos contradictorios y poco claros.
El aspecto, la presencia y el enfoque no están de acuerdo con lo que relata (sin angustia, voz fuerte y firme,
y sin compromiso emocional). Asegura enfáticamente que no puede retomar su trabajo, razón por la que
acude al Servicio de Urgencia, pidiendo un certificado dado que el día siguiente tiene que presentarse a un
proceso judicial por divorcio. Cuál es la impresión diagnóstica más probable en este paciente?
1. T. de Personalidad.
2. T. disociativo de personalidad (Personalidad Múltiple)
3. Simulación
4. T. Disociativo posttraumático
5. T. Facticio.
9. Muchacha de 19 años estudiante, sin antecedentes personales somáticos ni psiquiátricos relevantes
que es llevada a urgencias hospitalarias por su familia por ingesta masiva de pastillas hace una hora.
Siendo la paciente encontrada por su madre casualmente al volver a su casa antes de lo que tenía
previsto. Paciente refiere presentar desde unas 4 semanas atrás un cuadro de tristeza, anhedonia,
desesperanza, sentimientos de culpa y deseos de muerte, inhibición psicomotriz y enlentecimiento del
pensamiento, dificultad de concentración y rendimiento en los estudios, aislamiento social, hipersomnia
diurna y empeoramiento matutino de los síntomas. Así mismo la paciente refería tener la sensación de
que sus vecinos la espiaban, hablaban de ella cuando salía y se reían de ella lo que había incrementado
su angustia. No sabía el motivo por el que la gente se fijaba en ella pero estaba convencida de que no
eran imaginaciones suyas. Señale el tratamiento farmacológico en urgencias
1. Amitriptilina intravenosa.
2. Lavado gástrico y carbón activado.
3. Fluoxetina altas dosis por vía oral.
4. Diazepam intravenosa para sedar al paciente.
5. Haloperidol Intravenoso.
10. Mariano es un señor de 53 años de edad que acude a su consulta refiriendo sentir un gran malestar
desde hace ya unos años. Relaciona el malestar con una conducta que encuentra absurda pero que es
incapaz de evitar. Esta conducta que lleva a cabo siempre al llegar a su casa de vuelta del trabajo
consiste en accionar el pomo del baño del piso superior de la vivienda antes de hacer cualquier otra
cosa, incluso antes de saludar a su familia. Algunas veces ha intentado resistirse a hacerlo pero solo
ha conseguido angustiarse y demorar la conducta unos minutos. Éste comportamiento que Mariano
realiza de un modo incoercible y automático es lo que se denomina:
1. Comportamiento normal.
2. Obsesión.
3. Impulsión.
4. Compulsión.
5. Tic complejo
11. Tratamiento de primera línea para el trastorno obsesivo compulsivo.
1. Sertralina 100 a 200 mg PO AM diario
2. Paroxetina 60mg PO HS diaria.
3. Terapia Psicológica ambulatoria.
4. Todas son terapias de primera línea.
5. Ninguna es terapia de primera línea.

12. Profesor varón, de 40 años de edad, natural y procedente del Chota, quién desde hace 10 años
aproximadamente y de forma reiterada insiste en un reclamo a una familia que se apellida Belaunde sobre
una herencia que supuestamente le corresponde refiere en su reclamo judicial “Mis apellidos son Vela
Terry, pero para confundir a la opinión pública y a mí persona, el apellido del ex -presidente lo han escrito
con “B” y le han añadido el “unde por lo que las herencias del presidente de la República me
corresponderían”. ¿Señale Ud. cuál es su hipótesis diagnóstica más probable? Escoja una opción
1. Demencia de Alzheimer.
2. Trastorno orgánico Cerebral.
3. Delirium Crónico.
4. Trastorno de ideas delirantes persistentes.
5. Demencia mixta vascular y senil.
13. Caso clínico. Un estudiante varón de 20 años de edad, previamente sano, se ha estado comportando de
forma cada vez más extraña. A veces parece enfadado, comenta a sus amigos que está siendo seguido
por la policia y por servicios secretos, a veces ríe solo sin motivo. Últimamente se preocupa más por sus
pensamientos y su rendimiento académico ha bajado. Cuando fue a consulta estaba inquieto y asustado.
Decía que escuchaba voces que comentaban sus acciones. Además piensa que le están robando sus
pensamientos y que en la televisión hacen referencia a él, estos síntomas inician según sus padres hace
dos meses y han ido empeorando. Estos son parte de síntomas del criterio A para esquizofrenia según el
DSM – IV – TR, excepto
1. Disfunción social y ambiental
2. Alucinaciones
3. Delirios
4. Comportamiento desorganizado o catatónico
5. Síntomas negativos como aplanamiento afectivo.
14. Una mujer de 23 años, sin historia de abuso de alcohol, refiere episodios matinales de temblor, ansiedad,
taquicardia y sudoración. Al realizar el examen físico se evidencia dermografía negativa y presencia de
rinitis inespecífica.
¿Cuál de los siguientes exámenes tiene mayor probabilidad de estar alterado?

1. Amonemia
2. Glicemia
3. Potasio sérico
4. T3, T4, TSH
5. Pesquisa de drogas en orina

15. El tratamiento farmacológico completo de un paciente con un primer episodio de depresión mayor, sin
síntomas psicóticos, debe realizarse por un tiempo MINIMO de:
1. 2 semanas
2. 2 meses
3. 5 meses
4. 6 meses
5. 24 meses
16. Una estudiante universitaria de 19 años acude a la consulta acompañada por sus padres refierendo
sentirse en los últimos dos meses progresivamente más asténica, con pérdida de apetito y de peso y
con mayores dificultades para concentrarse en los estudios. En la anamnesis también destaca que ha
perdido interés en salir con las amigas, presenta ideas de muerte con ideación autolítica y cogniciones
pesimistas de futuro. Su peso es el 90% del considerado ideal por edad y género. No presenta fobia
ponderal ni distorsión de la imagen corporal. El diagnóstico más adecuado es:
1. Anorexia nerviosa.
2. Trastorno depresivo leve
3. Trastorno Depresivo grave con Psicosis
4. Distimia.
5. Trastorno depresivo grave.

17. Señale cual de los siguientes tratamientos NO está indicado para el tratamiento crónico del trastorno
Psicótico Breve.
1. Quetiapina
2. Olanzapina
3. Risperidona.
4. Metilfenidato.
5. Aripiprazol.

18. Señalar cual es el trastorno donde el paciente tiende a acudir al médico frecuentemente por el simple
hecho de buscar atención médica que incluso llega al hecho de enfermarse voluntariamente.
1. Trastorno de personalidad narcisista.
2. Trastornos disociativos.
3. Trastorno hipocondriaco.
4. Trastorno límite de la personalidad.
5. Trastorno facticio.
19. Qué fármaco podría causar un brote maniacal en un tratamiento de un paciente con depresión crónica de
40 años de evolución y tratamiento antidepresivo contínuo en los últimos 30 años ?
1. Ninguno podría.
2. Bupropion (Wellbutrim)
3. Todos podrían.
4. Sertralina.
5. Venlafaxina.
20. Mujer de 60 años de edad que presenta ideación suicida, cortes profundos en muñecas, obsesiones sobre
contaminación, olvidos frecuentes, baja de peso, traída agitada por sus familiares, insulta en el cuarto de
emergencia y le dice a usted que por favor no le mate y se muestra entre agresiva y confusa, cuál de estos
diagnósticos es el menos probable que sea la causa de esta sintomatología?
1. TOC descompensado
2. Delirium.
3. Trastomo adaptativo depresivo.
4. Episodio de depresión mayor.
5. Demencia tipo Alzheimer.
21. Mujer de 26 años sin antecedentes psiquiátricos previos, acude a Urgencias traída por sus padres que
explican que desde hace unos días está hiperactiva, nerviosa, insomne. Explican que revisa obsesivamente
la instalación eléctrica de casa en busca de cámaras de vídeo y micrófonos. En la entrevista nos dice que
le están vigilando desde la Policía pues ella es una enviada galáctica con poderes especiales. ¿Cuál de las
siguientes opciones NO debe considerarse como diagnóstico diferencial?
1. Trastorno obsesivo compulsivo.
2. Esquizofrenia.
3. Episodio maníaco.
4. Psicosis por consumo de sustancias tóxicas.
5. Tumor cerebral.
22. Referido a las formas clínicas de la esquizofrenia ¿cuál de las siguientes afirmaciones es cierta?
1. La forma CATATÓNICA se presenta con sintomatología pobre, predominio de delirios y suele ser
de inicio temprano.
2. En la forma HEBERFRÉNICA existe predominio de ateraciones psicomotrices, negativismo y
conductas esteriotipadas.
3. La presencia de estados de exaltación anímica, maniforme con extravagancias es propia de las
formas SIMPLES.
4. La forma PARANOIDE presenta predominantemente trastornos del pensamiento y trastornos
sensoperceptivos.
5. Los síntomas obsesivos y compulsivos asociados a disgregación progresiva son habitualmente en
la forma, ESQUIZOTÍPICA.
23. Cuál de las siguientes afirmaciones NO ES CIERTA para el trastorno bipolar?
1. Se presenta con la sucesión de fases depresivas y maníacas, aunque pueden presentarse
intervalos de normalidad.
2. La TEC (terapia electroconvulsiva) es indicación de primera linea en el trastorno bipolar durante el
embarazo.
3. La herencia admitida en el trastorno bipolar es de tipo autosómico recesiva con penetrancia
incompleta.
4. El tratamiento del trastorno bipolar con carbonato de litio está indicado tanto en fase maníaca como
para una terapéutica profiláctica.
5. La presencia de delirios organizados excluye el diagnóstico de trastorno bipolar.

24. La presencia de síntomas neurológicos sin explicación médica se explicaría mejor por cuál de las siguientes
enfermedades psiquiátricas?
1. Trastorno facticio.
2. Simulación.
3. Trastorno por conversión.
4. Trastorno somatomorfo
5. Todas son correctas

25. Los tres componentes básicos para el diagnóstico de autismo son:


1. Mala Interacción social, retraso mental y falta de juego espontáneo.
2. Imposibilidad de comunicar sentimientos, retraso mental y retraso en el lenguaje.
3. Conductas restrictivas, pobre interacción social y retraso o ausencia en el desarrollo del lenguaje.
4. Déficit cognitivo, pobre interacción social y retraso en el lenguaje.
5. Afectación social/laboral; retraso en el lenguaje y que no sea causado por enfermedad médica.

26. Qué hipótesis neurobiológica no se relaciona con la depresión de una forma relevante:
1. Hipótesis hormonal relacionada con control de la FSH y la LH
2. Hipótesis genética relacionada con alteraciones génicas en la síntesis del recaptador de serotonina.
3. Hipótesis de Estrés relacionada con alteración en la plasticidad cerebral como alteraciones en el
CREB y otro tipo de quinasa y activadores de secuencias repetitivas.
4. Hipótesis de alteración dopaminérgica.
5. Hipótesis aminérgica.
27. Mujer de 47 años sin antecedentes psiquiátricos que es hospitalizada para estudio de metrorragias y a los
5 días de su ingreso es informada del diagnóstico de neoplasia uterina con metástasis. Se consulta a
Psiquiatría porque 24 horas después muestra tristeza y llanto frecuente, refiere ideas de muerte y presenta
insomnio. En la evaluación psiquiátrica no se recoge ningún antecedente psiquiátrico y la exploración
detecta elevada ansiedad y desesperanza en relación con las consecuencias de su enfermedad neoplásica.
El diagnóstico más probable es:

1. Trastorno esquizoafectivo.
2. Distimia.
3. Trastorno de ansiedad generalizada.
4. Trastorno por estrés postraumático.
5. Trastorno adaptativo.
28. Un señor de 60 años, empleado de comercio, acude a urgencias acompañado de su familia; éstos cuentan
que el día anterior, de forma bastante brusca, comenzó a decir cosas raras, a no responder a lo que le
preguntaban y a mostrar se confuso incluso respecto de su propio nombre. Efectivamente en la exploración
parece no entender lo que se le pregunta, no recuerda nada de lo que le ha pasado y no sabe ni el día, ni
el lugar en el que está. El diagnóstico sería:

1. Psicosis breve.
2. Delirium.
3. Alzheimer.
4. Esquizofrenia.
5. Trastorno de ansiedad
29. ¿En cuál de los siguientes casos en una mujer embarazada la terapia electroconvulsivante NO es un
tratamiento de primera elección?:

1. Esquizofrenia paranoide.
2. Psicosis reactiva.
3. Depresión mayor con ideación suicida.
4. Trastorno esquizotípico de la personalidad.
5. Trastorno de ideas delirante persistentes crónico.

30. Cuál de estos fármacos no es un ISRS?


1. Venlafaxina
2. Sertralina
3. Fluoxetina
4. Paroxetina
5. Escitalopram
31. Cuánto tiempo cómo máximo debe durar la sintomatología de un Trastorno Esquizofreniforme?
1. 2 semanas
2. 1 mes
3. 3 meses
4. 6 meses
5. Es crónico “de por vida”

32. Cuál son los síntomas cardinales en un cuadro depresivo?


1. Hipotimia y anhedonia
2. Astenia y bajo estado de ánimo.
3. Ideación suicida y baja de peso,
4. Alteraciones en apetitio y sueño.
5. Baja autoestima y tristeza.
33. Cuál de estos fármacos no es un eutimizante?
1. Olanzapina
2. Venlafaxina
3. Quetiapina
4. Litio
5. Acd. Valpróico
34. Paciente de 15 años de edad, acude a consulta ambulatoria general, acompañada de sus padres, refieren
que han encontrado en su habitación una carta de despedida al parecer con objetivos suicidas, al entrevistar
a la paciente ella niega deseos suicidas sin embargo se muestra muy renuente y negativa durante la
entrevista, cuál es su siguiente paso como médico General en esta situación?
1. Explicar a los padres que probablemente su hija necesite atención Psicológica dado que podría
estar atravesando una depresión.
2. Iniciar tratamiento antidepresivo como Fluoxetina.
3. No dar ninguna explicación especial dado que esto es algo frecuente y la paciente esta manipulando
la situación.
4. Enviar a emergencia psiquiátricas a la paciente para probable internamiento.
5. Llamar a una nueva consulta en un mes y explicar a los padres que deben estar atentos de la
paciente.
35. Cuál de estos fármacos antidepresivos no presenta como efecto colateral disminución de la líbido en los
hombres?
1. Bupropion XL.
2. Venlafaxina.
3. Fluoxetina.
4. Sertralina.
5. Mirtazapina.
36. Que es el vaginismo?
1. Aumento de flora anormal en la mucosa vaginal.
2. Contracción involuntaria de las paredes musculares vaginales.
3. Infección vaginal gonocócica.
4. Resequedad de las paredes vaginales.
5. Infección vaginal indeterminada.
37. Que es el Voyeurismo?
1. Placer a través del frotamiento de órganos genitales con objetos o personas.
2. Placer con uso de correas en el acto sexual.
3. Placer en la sensación de traicionar a la pareja sentimental.
4. Placer al sentir dolor o mediante juegos de dominación durante el acto sexual.
5. Placer al ver a otras personas en situaciones eróticas.
38. Estos son criterios para trastorno de ansiedad Generalizada excepto:
1. Ansiedad y preocupación excesivas
2. Inquietud o impaciencia; fatigabilidad fácil.
3. Tensión muscular, irritabilidad
4. El Tiempo mínimo de todos los síntomas es de un mes.
5. La sintomatología no debe ser causada por drogas.
39. Cuál es el principal diagnóstico diferencial en una fuga disociativa ?
1. Convulsión.
2. Demencia de Alzheimer
3. Delirium
4. Intoxicación con drogas
5. Depresión Psicótica.
40. Qué es un heteroreceptor en la neurotransmisión ?
1. Es un receptor diferente al neurotransmisor que lo activa.
2. En un receptor capaz de activarse por diferentes Neurotransmisores.
3. Es un receptor que se sitúa en la membrana de una neurona de diferente clase en relación al ligando
que lo estimula.
4. Es un receptor atípico el cual presenta una heterogenicidad en relación a la activación de la protenia
G.
5. Es un receptor que presenta dos subunidades diferentes la una con la otra.
41. PREGUNTA BONUS: Cuál es el rango normal terapeútico del lítio en Sangre? (mEq/L)
CASO 1

DF: Paciente femenino de 32 años de edad, nacida y residente en Quito, instrucción secundaria completa, casada tiene 2
hijos (3 y 5 años) resto de datos de filiación sin relevancia.
MC: Aislamiento
EA: Paciente es traída por su esposo dado que hace 4 días prácticamente no quiere levantarse de la cama ni salir de la
casa
Cuando a la paciente en la entrevista se le pregunta que nos comente lo que sucede, la paciente responde así:
“Doctor no sé lo que me pasa, quiero solo quedarme acostada en la cama, si me levanto es solo para atender a mis hijos
y ya no siento energía como antes, paso llorando y triste, las noches ya no duermo me paso en vela casi toda la noche y
en la mañana estoy super cansada, a veces me dan unas angustias que quiero salir corriendo y el corazón se me quiere
salir, ayúdeme doctor creo que me voy a volver loca, ya ni siquiera puedo serle útil a mi marido en la cama y eso me pone
peor, el otro día que salí al mercado me daban hasta ganas de lanzarme contra un carro para que me atropelle y terminar
con todo esto, a veces siento sensaciones que me hablan y me dicen que me mate, me he estado tomando unas gotas
de valeriana en la noche porque no puedo dormir”.
1. Cuál es tu impresión diagnóstica?
a) Distimia
b) Trastorno adaptativo tipo depresivo
c) Trastorno depresivo grave con síntomas psicóticos
d) Trastorno Psicótico agudo
e) Trastorno Esquizoafectivo
2. Cuáles son las preguntas más importantes que realizarías para llegar a tu diagnóstico que respondiste en la
pregunta 1.
a) Tiempo de Evolución
b) Antecedentes Genéticos - Familiares
c) Razón por que se santigua?
d) Egosintonismo o egodistonismo
e) Todas son igual de importantes
3. Si tú eres médico rural en un subcentro de salud, cual sería tu conducta a seguir con esta paciente?
a) Inicio tratamiento con Antidepresivos y antipsicóticos.
b) Transfiero a Paciente a Hospital más cercano.
c) Inicio tratamiento con benzodiacepinas y Antidepresivos.
d) Inicio tratamiento con antidepresivos, antipsicóticos y con benzodiacepinas.
e) Le aconsejo que tome una cita con Psiquiatría.
4. Una enfermedad en que los intentos de suicidio se presenten sin planificación.
a) Trastorno límite de la personalidad.
b) Cualquier psicosis con alucinaciones que estimulen al suicidio
c) Intoxicación por drogas
d) En todas las opciones se puede presentar
e) En ninguna de las opciones se puede presentar
5. Cuál es el tratamiento farmacológico de mantenimiento más probable en este caso?
a) Clonazepam 2mg PO TID por 2 meses y risperidona 3mg HS por 1 año.
b) Carbonato de Litio 300mg TID de por vida.
c) Sertralina 50mg PO AM y Risperidona 3mg por un año.
d) Clonazepam 2mg PO TID y Sertralina 50mg PO AM por un año.
e) Clonazepam 2mg PO TID y Sertralina 50mg PO AM por un año.

CASO 2

DF: Se trata de un paciente de 21 años de edad masculino, nacido en Riobamba residente en Quito, estudia odontología,
no tiene hijos vive con sus padres. Resto de datos de filiación sin importancia.
MC: Cambios en la conducta de su hijo,
Al iniciar la entrevista ingresan los dos padres con el paciente se realiza una pregunta abierta como: “En que les puedo
ayudar¨ MADRE: Doctor venimos por nuestro hijo, hace aproximadamente 2 meses ha iniciado con comportamientos
extraños como santiguarse varias veces durante el día sin ninguna razón, en un inicio pensábamos que era porque estaba
en el grupo de oración que nuestra familia acude, pero luego comenzó a ser ya exagerado, sus amigos le comenzaron a
molestar y mi hijo se alejó de ellos.
Luego hablo con el muchacho directamente y le pregunto si me podría comentar que sucede, el responde así: Doctor ya
no se que hacer tengo una necesidad de santiguarme todo el tiempo, en un inicio eran unas pocas veces y no me molestaba
pero ahora ya no se que hacer, esto me desespera, quiero controlarlo y no puedo, siento que algo me está pasando, me
provoca demasiada ansiedad cuando trato de controlarme. Pregunto si esto ha afectado en algo su desenvolvimiento
académico: El responde que no pero que le da vergüenza estar en clase y tener que santiguarse. Pregunto si ha afectado
su área social, el responde que sus amigos le molestan y creen que está loco y que esto ha hecho que ya casi no salga
con amigos. Pregunto si se ha sentido triste o con poca energía y niega estos y otros síntomas depresivos.
6. Cuál es tu impresión diagnóstica? (un solo diagnóstico)
a) Trastorno de Ansiedad no especificada.
b) Trastorno de control de impulsos
c) TOC
d) Trastorno de ideas delirantes persistentes
e) Trastono de personalidad múltiples
7. Cuál es la pregunta más importante que realizarías para llegar a tu diagnóstico. (UNA SOLA PREGUNTA)
a) Tiempo de Evolución
b) Antecedentes Genéticos - Familiares
c) Razón por que se santigua?
d) Egosintonismo o egodistonismo
e) Todas son igual de importantes
8. Cuál sería el tratamiento de primera elección en este caso y por cuánto tiempo?
a) Risperidona 3mg HS por 1 año.
b) Risperidona 3mg HS de por vida.
c) Sertralina 50mg PO AM por un año.
d) Sertralina 200mg PO AM de por vida
e) Sertralina 50mg + Risperidona 3mg por un año.

STUDY CASE 3

Gina was sure she was having a heart attack. She woke up shaking and trembling with a pain in her chest. Her heart
seemed as though it would leap right out of her chest, it was beating so fast. She had no idea what was happening to her.
She thought that she may be dying, so she went to the ER.(Emergency Room). At the ER, the doctor did an EKG and took
some blood work. However, all the results were negative. The doctor asked Gina to speak to the hospital social worker,
and she agreed. After an intimate and confidential conversation, the social worker learned that Gina had recently broken
up with her long time boyfriend, and had also lost her job all in the same time period. The social worker suggested that Gina
should visit a psychiatrist. The psychiatrist ask about the incident and he got the next information: The patient had a extrem
sence of anxiety, he describe like been in the edge, a feeling of losing control of himself, hands sweating, this sensations
last a round of 10 minutes, she had 3 similar episodes before he decide to visit the Psychiatrist.

9. What is the most probable Diagnosis?


a) Generalized Anxiety Disorder.
b) Depression with anxiety features.
c) Obsessive compulsive disorder
d) Panic Disorder
e) Social Fobia
10. First line pharmacological Treatment in this case?
a) Sertraline 50mg PO AM
b) Pregabaline 150mg PO BID
c) Fluoxetine 20mg PO AM
d) All are first line options
e) Escitalopram 10mg PO AM

CASO 4
Paciente de 34 años, abogado, soltero, trabaja de forma independiente, dificultad de mantener un trabajo contínuo o
estable, ha tratado de trabajar en relación de dependencia pero siempre ha tenido problemas con sus jefes y sus
compañeros al parecer por extrema desconfianza que el tiene a las personas, sus dos internamientos previso tuvieron
como causa aparente conductas agresivas en contra de su familia especialmente pero también se ha presentado esta
agresividad en contra de otras personas cercanas, la causa aparente de la agresividad es por pensamientos que su familia
está en contra de él, que quieren aprovecharse de el en todo sentido, incluso que le espían en las noches.
11. Cuál es su impresión diagnóstica?
a) T. Psicótico Breve
b) T. de Ideas Delirantes Persistentes
c) T. Esquizofreniforme
d) Esquizofrenia
e) Trastorno Esquizoafectivo
12. Cuál es el tratamiento en este caso y por cuánto tiempo?
a) Risperidona 25mg IM Cada mes de por vida.
b) Olanzapina 10mg PO AM por un año.
c) Quetiapina 100mg PO BID de por vida.
d) Fluoxetina 20mg PO AM por un año
e) Fluoxetina 20mg PO de por vida.
________________________________________________________________________________________

13. Señalar el trastorno de la personalidad en el que es más frecuente y grave el aislamiento social:
a) Trastorno narcisista de la personalidad.
b) Trastorno paranoide de la personalidad.
c) Trastorno antisocial de la personalidad.
d) Trastorno esquizoide de la personalidad.
e) Trastorno obsesivo de la personalidad

14. Paciente de 53 años, lavandera, abandonada por marido alcohólico. Tiene 6 hijos, el mayor de los cuales es
alcohólico y los dos menores tienen serias dificultades en el colegio.
Desde los 20 años presenta episodios de vómitos, dolores abdominales y musculares, intolerancia a diversos
alimentos, palpitaciones, mareos, dificultad para deglutir, visión borrosa y menstruación dolorosa.
Consulta por prurito anal y dolor rectal, sin causa orgánica demostrada por exámenes.
Se aprecia triste y se queja de insomnio de conciliación. Su personalidad es introvertida, pesimista, ansiosa y
demandante de afecto.
El diagnóstico más probable es:
a) trastorno de ansiedad generalizada
b) episodio depresivo mayor, leve, con somatizaciones
c) trastorno de somatización.
d) trastorno hipocondriaco
e) trastorno de personalidad limítrofe
15. Escolar de 8 años, con buena conducta previa, presenta cambios importantes desde hace tres meses desde la
muerte de su abuelo con quien vivía, baja en sus notas de escuela, mal humor, tendencia a aislarse socialmente,
conflictivo y agresivo con sus padres, se irrita fácilmente, inquieto y falto de concentración en el aula, muy poco
motivado en general. El diagnóstico más probable es:
a) trastorno de conducta disocial
b) síndrome de déficit atencional
c) trastorno depresivo
d) trastorno de conducta oposicionista desafiante
e) síndrome ansioso
16. A diferencia de la angustia del Trastorno de ansiedad generalizada, la angustia en la crisis de pánico se
caracteriza por:
a) ser egosintónica
b) ser egodistónica
c) relacionarse con eventos específicos
d) no tener motivo aparente
e) incluir expectación ansiosa
17. Qué vía dopaminérgica se encuentra alterada en la presencia de “Galactorrea” con el uso de antipsicóticos?
a) Nigroestraida
b) Tuberoinfundibular
c) Nigrohipofisiaria
d) Hipofiso límbica
e) Mesolímbica
18. Cuál es el tratamiento de elección en Trastorno Bipolar episodio maniaco durante el embarazo?
a) C. de litio.
b) Acd. Valpróico
c) TEC
d) Olanzapina
e) Carbamazepina
19. Mujer de 54 años, ex funcionaria pública de la salud, vive con pareja alcohólica. Paciente es diabética e hipertensa
con tratamientos irregulares en consultorios. No trabaja, vive de una pensión de invalidez. Consume alcohol desde
hace aproximadamente 30 años, con múltiples tratamientos en servicios psiquiátricos, unidades especializadas,
grupos sociales. Llega al Servicio de Urgencia con nueva crisis de ingesta etílica, lo que ocurre con frecuencia,
en estado de ebriedad, llorosa y arrepentida, jurando que dejará de beber esta vez. Marcha atáxica y problemas
con la pareja estando bajo los efectos del alcohol. Manifiesta mucha angustia, sudor, taquicardia, polidipsia y
polifagia. Trastornos del Sueño. Dice que “no duerme hace dos días”. Bebiendo vino en forma intermitente.
Cuál es la conducta a seguir si recibes esta paciente en tu rural (Trabajas en Hospital Provincial básico):
a) Evaluar sobre conductas de abstinencia y tolerancia para transferir a Centro de especialidad en
Addicciones
b) Atender de forma ambulatoria iniciar tratamiento y llamar a la paciente en un mes.
c) Evaluar con exámenes de laboratorio de rutina y específicos por sus síntomas.
d) Ingresar al Hospital básico para evaluación y tratamiento.
e) Transferir de emergencia a Hospital de alta complejidad de tercer nivel.
20. Frente a un delirio celotípico hay que sospechar la existencia de:
a) SIDA.
b) Alcoholismo.
c) Demencia.
d) Personalidad esquizoide.
e) Depresión Grave
21. Señalar cuáles de los siguientes síntomas de la esquizofrenia constituye un factor de mal pronóstico:
a) Pródromo breve.
b) Ausencia de síntomas depresivos.
c) Inicio tardío.
d) Ausencia de síntomas obsesivos.
e) Presencia de Síntomas negativos
22. Señalar en que trastorno de la personalidad se observa la ausencia de remordimientos y la incapacidad para
planificar el futuro:
a) Trastorno paranoide de la personalidad.
b) Trastorno límite de la personalidad.
c) Trastorno antisocial de la personalidad.
d) Trastomo de la personalidad por evitación.
e) Trastorno Dependiente de la Personalidad.
23. Hombre de 26 años, soltero, que es traído a consulta por su familia por llevar 3 meses negándose a salir se su
casa. La razón que aduce el paciente es el convencimiento de que tiene la mandíbula asimétrica y la cara torcida.
Según refiere esta situación es progresiva y cada vez se ve mas deforme cuando se observa en el espejo. Se
avergüenza de su aspecto, por lo que no quiere salir, se angustia mucho cuando ve su imagen y no puede dejar
de pensar todo el día en su deformidad. Ha consultado con varios cirujanos maxilofaciales pero éstos le dicen
que no presenta asimetría facial y lo remiten al psiquiatra. El diagnóstico del paciente es:
a) Trastorno depresivo mayor con ideas delirantes incongruentes con el estado de ánimo.
b) Trastorno obsesivo compulsivo.
c) Esquizofrenia paranoide.
d) Trastorno dismórfico corporal
e) Trastorno de Somatización.
24. Una paciente presenta una larga historia de múltiples síntomas físicos: pérdida de memoria, dolor de cabeza,
mareos, vómitos, dolor genital, dolor en extremidades, distensión abdominal e irregularidades menstruales. Los
diversos exámenes médicos han descartado enfermedad médica alguna. ¿Cuál de los siguientes cuadros es más
probable que padezca?
a) Trastorno conversivo.
b) Trastorno hipocondríaco.
c) Trastorno de somatización.
d) Trastomo disociativo.
e) Trastorno de Ideas delirantes Persistentes
25. Mujer de 17 años, estudiante, vive con sus padres y hermanos. Angustiada llora constantemente, decaída,
desanimada, ha dejado de salir a fiestas y ya no comparte con amigos. La noche anterior ingiere cantidad no precisada
de tabletas “para los nervios” que tomaba su madre. No es primera vez que lo hace, ésta es la tercera vez y siempre
ha sido luego de un problema. Refiere estar arrepentida por lo que hizo.
Cuál es el probable trastorno de personalidad que acompaña a esta mujer que acude con criterios para depresión?
a) T. de personalidad Histriónica.
b) T. de personalidad Ciclitímica
c) T. de personalidad Obsesiva
d) T. de personalidad Esquizotípica
e) T. de personalidad Limítrofe
26. Paciente sexo masculino 30 años, casado. Acude con una carpeta con documentos bajo el brazo, buen contacto,
limpio y bien vestido. Relata que hace 6 meses que está con un cuadro depresivo severo, que ha pasado por
múltiples tratamientos y diferentes médicos y no ha mejorado. Con licencia médica desde el inicio de su
enfermedad, la cual se inició tras separación conyugal (por segunda vez) porque, “ella no me comprendía”.
Asegura ser un excelente trabajador y esposo. Niega consumo de drogas y/o alcohol.Al tratar de establecer una
sintomatología precisa del momento actual, aporta datos contradictorios y poco claros. El aspecto, la presencia y
el enfoque no están de acuerdo con lo que relata (sin angustia, voz fuerte y firme, y sin compromiso emocional).
Asegura enfáticamente que no puede retomar su trabajo, razón por la que acude al Servicio de Urgencia, pidiendo
un certificado dado que el día siguiente tiene que presentarse a un proceso judicial por divorcio. Cuál es la
impresión diagnóstica más probable en este paciente?
a) T. de Personalidad.
b) T. disociativo de personalidad (Personalidad Múltiple)
c) Simulación
d) T. Disociativo posttraumático
e) T. Facticio.
27. Muchacha de 19 años estudiante, sin antecedentes personales somáticos ni psiquiátricos relevantes que es
llevada a urgencias hospitalarias por su familia por ingesta masiva de pastillas hace una hora. Siendo la
paciente encontrada por su madre casualmente al volver a su casa antes de lo que tenía previsto. Paciente
refiere presentar desde unas 4 semanas atrás un cuadro de tristeza, anhedonia, desesperanza, sentimientos
de culpa y deseos de muerte, inhibición psicomotriz y enlentecimiento del pensamiento, dificultad de
concentración y rendimiento en los estudios, aislamiento social, hipersomnia diurna y empeoramiento matutino
de los síntomas. Así mismo la paciente refería tener la sensación de que sus vecinos la espiaban, hablaban
de ella cuando salía y se reían de ella lo que había incrementado su angustia. No sabía el motivo por el que
la gente se fijaba en ella pero estaba convencida de que no eran imaginaciones suyas. Señale el tratamiento
farmacológico en urgencias
a) Amitriptilina intravenosa.
b) Lavado gástrico y carbón activado.
c) Fluoxetina altas dosis por vía oral.
d) Diazepam intravenosa para sedar al paciente.
e) Haloperidol Intravenoso.
28. Mariano es un señor de 53 años de edad que acude a su consulta refiriendo sentir un gran malestar desde
hace ya unos años. Relaciona el malestar con una conducta que encuentra absurda pero que es incapaz de
evitar. Esta conducta que lleva a cabo siempre al llegar a su casa de vuelta del trabajo consiste en accionar
el pomo del baño del piso superior de la vivienda antes de hacer cualquier otra cosa, incluso antes de saludar
a su familia. Algunas veces ha intentado resistirse a hacerlo pero solo ha conseguido angustiarse y demorar
la conducta unos minutos. Éste comportamiento que Mariano realiza de un modo incoercible y automático es
lo que se denomina:
a) Comportamiento normal.
b) Obsesión.
c) Impulsión.
d) Compulsión.
e) Tic complejo
29. Tratamiento de primera línea para el trastorno obsesivo compulsivo.
a) Sertralina 100 a 200 mg PO AM diario
b) Paroxetina 60mg PO HS diaria.
c) Terapia Psicológica ambulatoria.
d) Todas son terapias de primera línea.
e) Ninguna es terapia de primera línea.
30. Profesor varón, de 40 años de edad, natural y procedente del Chota, quién desde hace 10 años
aproximadamente y de forma reiterada insiste en un reclamo a una familia que se apellida Belaunde sobre
una herencia que supuestamente le corresponde refiere en su reclamo judicial “Mis apellidos son Vela Terry,
pero para confundir a la opinión pública y a mí persona, el apellido del ex -presidente lo han escrito con “B” y
le han añadido el “unde por lo que las herencias del presidente de la República me corresponderían”. ¿Señale
Ud. cuál es su hipótesis diagnóstica más probable? Escoja una opción
a) Demencia de Alzheimer.
b) Trastorno orgánico Cerebral.
c) Delirium Crónico.
d) Trastorno de ideas delirantes persistentes.
e) Demencia mixta vascular y senil.
31. Caso clínico. Un estudiante varón de 20 años de edad, previamente sano, se ha estado comportando de forma
cada vez más extraña. A veces parece enfadado, comenta a sus amigos que está siendo seguido por la policia y
por servicios secretos, a veces ríe solo sin motivo. Últimamente se preocupa más por sus pensamientos y su
rendimiento académico ha bajado. Cuando fue a consulta estaba inquieto y asustado. Decía que escuchaba voces
que comentaban sus acciones. Además piensa que le están robando sus pensamientos y que en la televisión
hacen referencia a él, estos síntomas inician según sus padres hace dos meses y han ido empeorando. Estos
son parte de síntomas del criterio A para esquizofrenia según el DSM – IV – TR, excepto
a) Disfunción social y ambiental
b) Alucinaciones
c) Delirios
d) Comportamiento desorganizado o catatónico
e) Síntomas negativos como aplanamiento afectivo.
32. Una mujer de 23 años, sin historia de abuso de alcohol, refiere episodios matinales de temblor, ansiedad,
taquicardia y sudoración. Al realizar el examen físico se evidencia dermografía negativa y presencia de rinitis
inespecífica.
¿Cuál de los siguientes exámenes tiene mayor probabilidad de estar alterado?
1. Amonemia
2. Glicemia
3. Potasio sérico
4. T3, T4, TSH
5. Pesquisa de drogas en orina

33. El tratamiento farmacológico completo de un paciente con un primer episodio de depresión mayor, sin síntomas
psicóticos, debe realizarse por un tiempo MINIMO de:
a) 2 semanas
b) 2 meses
c) 5 meses
d) 6 meses
e) 24 meses
34. Una estudiante universitaria de 19 años acude a la consulta acompañada por sus padres refierendo sentirse
en los últimos dos meses progresivamente más asténica, con pérdida de apetito y de peso y con mayores
dificultades para concentrarse en los estudios. En la anamnesis también destaca que ha perdido interés en
salir con las amigas, presenta ideas de muerte con ideación autolítica y cogniciones pesimistas de futuro. Su
peso es el 90% del considerado ideal por edad y género. No presenta fobia ponderal ni distorsión de la imagen
corporal. El diagnóstico más adecuado es:
a) Anorexia nerviosa.
b) Trastorno depresivo leve
c) Trastorno Depresivo grave con Psicosis
d) Distimia.
e) Trastorno depresivo grave.
35. Señale cual de los siguientes tratamientos NO está indicado para el tratamiento crónico del trastorno Psicótico
Breve.
a) Quetiapina
b) Olanzapina
c) Risperidona.
d) Metilfenidato.
e) Aripiprazol.

36. Señalar cual es el trastorno donde el paciente tiende a acudir al médico frecuentemente por el simple hecho
de buscar atención médica que incluso llega al hecho de enfermarse voluntariamente.
a) Trastorno de personalidad narcisista.
b) Trastornos disociativos.
c) Trastorno hipocondriaco.
d) Trastorno límite de la personalidad.
e) Trastorno facticio.
37. Qué fármaco podría causar un brote maniacal en un tratamiento de un paciente con depresión crónica de 40 años
de evolución y tratamiento antidepresivo contínuo en los últimos 30 años ?
1. Ninguno podría.
2. Bupropion (Wellbutrim)
3. Todos podrían.
4. Sertralina.
5. Venlafaxina.
38. Mujer de 60 años de edad que presenta ideación suicida, cortes profundos en muñecas, obsesiones sobre
contaminación, olvidos frecuentes, baja de peso, traída agitada por sus familiares, insulta en el cuarto de
emergencia y le dice a usted que por favor no le mate y se muestra entre agresiva y confusa, cuál de estos
diagnósticos es el menos probable que sea la causa de esta sintomatología?
a) TOC.
b) Delirium.
c) Tumor cerebral.
d) Episodio de depresión mayor.
e) Demencia tipo Alzheimer.
39. Mujer de 26 años sin antecedentes psiquiátricos previos, acude a Urgencias traída por sus padres que explican
que desde hace unos días está hiperactiva, nerviosa, insomne. Explican que revisa obsesivamente la instalación
eléctrica de casa en busca de cámaras de vídeo y micrófonos. En la entrevista nos dice que le están vigilando
desde la Policía pues ella es una enviada galáctica con poderes especiales. ¿Cuál de las siguientes opciones NO
debe considerarse como diagnóstico diferencial?
a) Trastorno obsesivo compulsivo.
b) Esquizofrenia.
c) Episodio maníaco.
d) Psicosis por consumo de sustancias tóxicas.
e) Tumor cerebral.
40. Referido a las formas clínicas de la esquizofrenia ¿cuál de las siguientes afirmaciones es cierta?
a) La forma CATATÓNICA se presenta con sintomatología pobre, predominio de delirios y suele ser de inicio
temprano.
b) En la forma HEBERFRÉNICA existe predominio de ateraciones psicomotrices, negativismo y conductas
esteriotipadas.
c) La presencia de estados de exaltación anímica, maniforme con extravagancias es propia de las formas
SIMPLES.
d) La forma PARANOIDE presenta predominantemente trastornos del pensamiento y trastornos
sensoperceptivos.
e) Los síntomas obsesivos y compulsivos asociados a disgregación progresiva son habitualmente en la
forma, ESQUIZOTÍPICA.
41. Cuál de las siguientes afirmaciones NO ES CIERTA para el trastorno bipolar?
a) Se presenta con la sucesión de fases depresivas y maníacas, aunque pueden presentarse intervalos de
normalidad.
b) La TEC (terapia electroconvulsiva) es indicación de primera linea en el trastorno bipolar durante el
embarazo.
c) La herencia admitida en el trastorno bipolar es de tipo autosómico recesiva con penetrancia incompleta.
d) El tratamiento del trastorno bipolar con carbonato de litio está indicado tanto en fase maníaca como para
una terapéutica profiláctica.
e) La presencia de delirios organizados excluye el diagnóstico de trastorno bipolar.

42. La presencia de síntomas neurológicos sin explicación médica se explicaría mejor por cuál de las siguientes
enfermedades psiquiátricas?
a) Trastorno facticio.
b) Simulación.
c) Trastorno Conversivo.
d) Trastorno somatomorfo
e) Todas son correctas
43. Los tres componentes básicos para el diagnóstico de autismo son:
a) Mala Interacción social, retraso mental y falta de juego espontáneo.
b) Imposibilidad de comunicar sentimientos, retraso mental y retraso en el lenguaje.
c) Conductas restrictivas, pobre interacción social y retraso o ausencia en el desarrollo del lenguaje.
d) Déficit cognitivo, pobre interacción social y retraso en el lenguaje.
e) Afectación social/laboral; retraso en el lenguaje y que no sea causado por enfermedad médica.
44. Qué hipótesis neurobiológica no se relaciona con la depresión de una forma relevante:
a) Hipótesis hormonal relacionada con control de la FSH y la LH
b) Hipótesis genética relacionada con alteraciones génicas en la síntesis del recaptador de serotonina.
c) Hipótesis de Estrés relacionada con alteración en la plasticidad cerebral como alteraciones en el CREB y
otro tipo de quinasa y activadores de secuencias repetitivas.
d) Hipótesis de alteración dopaminérgica.
e) Hipótesis aminérgica.
45. Mujer de 47 años sin antecedentes psiquiátricos que es hospitalizada para estudio de metrorragias y a los 5 días
de su ingreso es informada del diagnóstico de neoplasia uterina con metástasis. Se consulta a Psiquiatría porque
24 horas después muestra tristeza y llanto frecuente, refiere ideas de muerte y presenta insomnio. En la
evaluación psiquiátrica no se recoge ningún antecedente psiquiátrico y la exploración detecta elevada ansiedad
y desesperanza en relación con las consecuencias de su enfermedad neoplásica. El diagnóstico más probable
es:

a) Trastorno esquizoafectivo.
b) Distimia.
c) Trastorno de ansiedad generalizada.
d) Trastorno por estrés postraumático.
e) Trastorno adaptativo.
46. Un señor de 60 años, empleado de comercio, acude a urgencias acompañado de su familia; éstos cuentan que
el día anterior, de forma bastante brusca, comenzó a decir cosas raras, a no responder a lo que le preguntaban y
a mostrar se confuso incluso respecto de su propio nombre. Efectivamente en la exploración parece no entender
lo que se le pregunta, no recuerda nada de lo que le ha pasado y no sabe ni el día, ni el lugar en el que está. El
diagnóstico sería:

a) Psicosis breve.
b) Delirium.
c) Alzheimer.
d) Esquizofrenia.
e) Trastorno de ansiedad
47. ¿En cuál de los siguientes casos en una mujer embarazada la terapia electroconvulsivante NO es un tratamiento
de primera elección?:

a) Esquizofrenia paranoide.
b) Psicosis reactiva.
c) Depresión mayor con ideación suicida.
d) Trastorno esquizotípico de la personalidad.
e) Trastorno de ideas delirante persistentes crónico.
48. Cuál de estos fármacos no es un ISRS?
a) Venlafaxina
b) Sertralina
c) Fluoxetina
d) Paroxetina
e) Escitalopram
49. Cuánto tiempo cómo máximo debe durar la sintomatología de un Trastorno Esquizofreniforme?
a) 2 semanas
b) 1 mes
c) 3 meses
d) 6 meses
e) Es crónico “de por vida”
50. Cuál son los síntomas cardinales en un cuadro depresivo?
a) Hipotimia y anhedonia
b) Astenia y bajo estado de ánimo.
c) Ideación suicida y baja de peso,
d) Alteraciones en apetitio y sueño.
e) Baja autoestima y tristeza.
51. Cuál de estos fármacos no es un eutimizante?
a) Olanzapina
b) Mirtazapina
c) Quetiapina
d) Litio
e) Acd. Valpróico
52. Paciente de 15 años de edad, acude a consulta ambulatoria general, acompañada de sus padres, refieren que
han encontrado en su habitación una carta de despedida al parecer con objetivos suicidas, al entrevistar a la
paciente ella niega deseos suicidas sin embargo se muestra muy renuente y negativa durante la entrevista, cuál
es su siguiente paso como médico General en esta situación?
a) Explicar a los padres que probablemente su hija necesite atención Psicológica dado que podría estar
atravesando una depresión.
b) Iniciar tratamiento antidepresivo como Fluoxetina.
c) No dar ninguna explicación especial dado que esto es algo frecuente y la paciente esta manipulando la
situación.
d) Enviar a emergencias psiquiátricas a la paciente para evaluación de especialidad y probable
internamiento.
e) Llamar a una nueva consulta en un mes y explicar a los padres que deben estar atentos de la paciente.
53. Cuál de estos fármacos antidepresivos no presenta como efecto colateral disminución de la líbido en los hombres?
a) Bupropion XL.
b) Venlafaxina.
c) Fluoxetina.
d) Sertralina.
e) Mirtazapina.
54. Qué es el vaginismo?
a) Aumento de flora anormal en la mucosa vaginal.
b) Contracción involuntaria de las paredes musculares vaginales.
c) Infección vaginal gonocócica.
d) Resequedad de las paredes vaginales.
e) Infección vaginal indeterminada.
55. Que es el Voyeurismo?
a) Placer a través del frotamiento de órganos genitales con objetos o personas.
b) Placer con uso de correas en el acto sexual.
c) Placer en la sensación de traicionar a la pareja sentimental.
d) Placer al sentir dolor o mediante juegos de dominación durante el acto sexual.
e) Placer al ver a otras personas en situaciones eróticas.
56. Estos son criterios para trastorno de ansiedad Generalizada excepto:
a) Ansiedad y preocupación excesivas
b) Inquietud o impaciencia; fatigabilidad fácil.
c) Tensión muscular, irritabilidad
d) El Tiempo mínimo de todos los síntomas es de un mes.
e) La sintomatología no debe ser causada por drogas.
57. Cuál es el principal diagnóstico diferencial en una fuga disociativa ?
a) Convulsión.
b) Demencia de Alzheimer
c) Delirium
d) Adicción a sustancias.
e) Depresión Psicótica.
58. Qué es un heteroreceptor en la neurotransmisión ?
a) Es un receptor diferente al neurotransmisor que lo activa.
b) En un receptor capaz de activarse por diferentes Neurotransmisores.
c) Es un receptor que se sitúa en la membrana de una neurona de diferente clase en relación al ligando que
lo estimula.
d) Es un receptor atípico el cual presenta una heterogenicidad en relación a la activación de la protenia G.
e) Es un receptor que presenta dos subunidades diferentes la una con la otra.
UNIVERSIDAD CENTRAL DEL ECUADOR – FACULTAD DE CIENCIAS MÉDICAS – CÁTEDRA DE PSIQUIATRÍA.

APELLIDOS Y NOMBRES……………………………………………FIRMA…………………………………………

CASO 1:
Se trata de un joven soltero de 25 años.
Motivo de consulta: El paciente fue persuadido por su hermano para que busque ayuda en un
hospital psiquiátrico porque tenía reacciones violentas e ideas que se quiere matar. Cinco semanas
antes había atacado a su madre sin advertencia y pegándole salvajemente, hasta que su hermano fue
en su ayuda. Durante las semanas siguientes tuvo una cantidad de explosiones de ira y varias
veces amenazó con quitarse la vida. Explicó el ataque a su madre diciendo que ella había tratado de
perjudicarlo y que había recibido instrucciones de una fuerza extraterrestre para pegarle, además cierra las cortinas por el miedo
a ser vigilado. Después del ataque se encerró en sí mismo, absorbido por sus propios pensamientos, y a menudo hablaba solo
aún cuando otras personas estuvieran alrededor. A veces su familia tenía la impresión que escuchaba voces que otras personas
no podían oir. Le dijo a su hermano mayor que tenía miedo de atacar a alguien, o de matarse; temía perder control de sus
propios actos.
Datos actuales: El paciente era un joven bien parecido y estaba vestido apropiadamente pero parecía que no se había bañado ya
varios días. Al ser examinado estaba tenso, hablaba rápido y en forma excitada. Tenía tendencia a agitar la mano sin razón
aparente. Su conversación estaba interrumpida por interpolaciones, y de vez en cuando se volvía incoherente e incomprensible.
Se sonreía superficial e inapropiadamente. En la expresión de sus afectos era cerrado y se enojaba al hablar de su madre. Decía
que ella lo hubiera querido muerto. Expresaba temor porque un poder extraterrestre llevaría su mente a otro planeta. Explicaba
que este poder controlaba sus pensamientos y le daba órdenes para lastimar a otras personas. Aparentemente el poder
extraterrestre hablaba sobre la situación de Ibrahim. Le decía que su madre lo quería ver muerto y le había dado instrucciones
para matarla. Los últimos días antes de la internación, Ibrahim consideró la idea de matarse para impedir que el poder extraño
tomara total
control de él.
1. Describa la alteración en el examen psiquiátrico que denotan las siguientes frases del caso ?
a. “ella había tratado de perjudicarlo y que había recibido instrucciones de una fuerza extraterrestre para pegarle:

b. “paciente fue persuadido por su hermano para que busque ayuda”

c. “no se había bañado ya varios días”

d. “el poder extraterrestre hablaba sobre la situación de Ibrahim”

2. Cuál es su Impresión diagnóstica?

3. Diagnósticos diferenciales principales.

a. .
b. .
c. .
d. .
e. .

4. Posible tratamientos según su impresión diagnóstica (dosis y posología)

a. .
b. .

8 de febrero de 2018
UNIVERSIDAD CENTRAL DEL ECUADOR – FACULTAD DE CIENCIAS MÉDICAS – CÁTEDRA DE PSIQUIATRÍA.

CASO 2
La paciente es una mujer divorciada de 52 años. Tiene una hija y es maestra en una escuela
primaria; nivel socioeconómico medio.

Motivo de consulta: Se quejaba de cansancio extremo al tener que permanecer despierta toda la
noche para impedir que alguien entrara a su casa a robar. Cuando tenía 35 años, comenzó a
lamentarse de ser el blanco de una discriminación proveniente de las autoridades de la escuela.
Decía que las maestras mayores intentaban perjudicarla debido a su diferente religión. Sentía que
la espiaban y que trataban de sacarla de su puesto de maestra. Algunos años después comenzó a
quejarse de que sus vecinos querían tener relaciones sexuales con ella. Los acusó de perseguirla y
de tenderle trampas para violarla. Hizo instalar trabas de seguridad en las puertas y ventanas y
nunca salía después del anochecer. Gradualmente se volvió ansiosa e irascible, y varias veces dio
parte a lo policía . Casi nunca se animaba a dormir de noche, por lo que se cansaba cada vez más.
Datos actuales: Durante la consulta la paciente parecía bastante normal. Su conversación era
coherente y acertada. Acusó a una cantidad de personas de querer hacerle daño y de que buscaban la
oportunidad de forzarla para que tenga relaciones sexuales con ellos. Negó tener alucinaciones. Le
desagradaba tener que hablar de estos problemas y parecía estar cansada por sus preocupaciones
persistentes. Estaba absolutamente convencida de que sus inquietudes eran reales. Aunque en
realidad nunca le había pasado nada, estaba segura de que tenía razón suficiente para creer que
podría pasarle. El examen físico sólo determinó obesidad leve.

1. Describa la alteración en el examen psiquiátrico que denotan las siguientes frases del caso ?
a. “Estaba absolutamente convencida de que sus inquietudes eran reales”

b. “Sentía que la espiaban”

c. “Los acusó de perseguirla y de tenderle trampas para violarla”

2. Cuál es su Impresión diagnóstica?

3. Diagnósticos diferenciales?

a. .
b. .
c. .
d. .
4. Realice un esquema gráfico de las vías dopaminérgicas en relación a los efectos colaterales que se presentan con los
antipsicóticos.

8 de febrero de 2018
UNIVERSIDAD CENTRAL DEL ECUADOR – FACULTAD DE CIENCIAS MÉDICAS – CÁTEDRA DE PSIQUIATRÍA.

1. Paciente sexo masculino 30 años, casado. Acude con una carpeta con documentos bajo el brazo, buen contacto,
limpio y bien vestido. Relata que hace 6 meses que está con un cuadro depresivo severo, que ha pasado por
múltiples tratamientos y diferentes médicos y no ha mejorado. Con licencia médica desde el inicio de su
enfermedad, la cual se inició tras separación conyugal (por segunda vez) porque, “ella no me comprendía”.
Asegura ser un excelente trabajador y esposo. Niega consumo de drogas y/o alcohol .Al tratar de establecer una
sintomatología precisa del momento actual, aporta datos contradictorios y poco claros. El aspecto, la presencia y
el enfoque no están de acuerdo con lo que relata (sin angustia, voz fuerte y firme, y sin compromiso emocional).
Asegura enfáticamente que no puede retomar su trabajo, razón por la que acude al Servicio de Urgencia, pidiendo
un certificado dado que el día siguiente tiene que presentarse a un proceso judicial por divorcio. Cuál es la
impresión diagnóstica más probable en este paciente?
1. T. de Personalidad.
2. T. disociativo de personalidad (Personalidad Múltiple)
3. Simulación.
4. T. por estrés postraumático.
5. T. Depresivo.
2. Profesor varón, de 70 años de edad, natural y procedente del Chota, quién desde hace 2 meses aproximadamente
y de forma reiterada insiste en un reclamo a una familia que se apellida Belaunde sobre una herencia que
supuestamente le corresponde refiere en su reclamo judicial “Mis apellidos son Vela Terry, pero para confundir a
la opinión pública y a mí persona, el apellido del ex -presidente lo han escrito con “B” y le han añadido el “unde
por lo que las herencias del presidente de la República me corresponderían”. El paciente presenta además
cambios conductuales hace aproximadamente un año que los familiares describen como que sale de la realidad
además de aislamiento y lo notan muy triste, antes del año el paciente era completamente normal ¿Señale Ud.
cuál es el diagnóstico menos probable en este caso.
1. Demencia de Alzheimer.
2. Trastorno orgánico Cerebral.
3. Esquizofrenia Paranoide.
4. Depresión con psicosis.
5. Demencia mixta vascular y senil.
3. El tiempo de tratamiento farmacológico para un paciente con diagnóstico de Esquizofrenia debe ser de:

1) 6 meses.
2) 12 meses.
3) 2 a 4 años.
4) de por vida.
5) depende de la respuesta.

4. Los tres componentes básicos para el diagnóstico de autismo son:


1. Mala Interacción social, retraso mental y falta de juego espontáneo.
2. Imposibilidad de comunicar sentimientos, retraso mental y retraso en el lenguaje.
3. Conductas restrictivas, pobre interacción social y retraso o ausencia en el desarrollo del lenguaje.
4. Déficit cognitivo, pobre interacción social y retraso en el lenguaje.
5. Afectación social/laboral; retraso en el lenguaje y que no sea causado por enfermedad médica.
5. Señale cuál de los siguientes tratamientos farmacológicos tiene mayor grado de evidencia y eficacia para el tratamiento de un paciente
con alcoholismo?
1. Olanzapina
2. Naltrexona Depot IM
3. Carbamazepina
4. Clonidina.
5. Complejo B más Tiamina.
6. En qué fase del sueño se presentan ondas alfa similares al estado de vigilia ?
1. REM
2. NREM Fase 1
3. NREM Fase 2
4. NREM Fase 3
5. NREM Fase 4

8 de febrero de 2018
UNIVERSIDAD CENTRAL DEL ECUADOR – FACULTAD DE CIENCIAS MÉDICAS – CÁTEDRA DE PSIQUIATRÍA.

7.
8. Señalar cuáles de los siguientes síntomas de la esquizofrenia constituye un factor de mal pronóstico:
1. Pródromo breve.
2. Ausencia de síntomas depresivos.
3. Inicio tardío.
4. Ausencia de síntomas obsesivos.
5. Presencia de Síntomas negativos
9. Señalar en que trastorno de la personalidad se observa la ausencia de remordimientos y la incapacidad para
planificar el futuro:
1. Trastorno paranoide de la personalidad.
2. Trastorno límite de la personalidad.
3. Trastorno antisocial de la personalidad.
4. Trastomo de la personalidad por evitación.
5. Trastorno Dependiente de la Personalidad.
10. Hombre de 26 años, soltero, que es traído a consulta por su familia por llevar 3 meses negándose a salir se su
casa. La razón que aduce el paciente es el convencimiento de que tiene la mandíbula asimétrica y la cara
torcida. Según refiere esta situación es progresiva y cada vez se ve mas deforme cuando se observa en el
espejo. Se avergüenza de su aspecto, por lo que no quiere salir, se angustia mucho cuando ve su imagen y no
puede dejar de pensar todo el día en su deformidad. Ha consultado con varios cirujanos maxilofaciales pero
éstos le dicen que no presenta asimetría facial y lo remiten al psiquiatra. El diagnóstico del paciente es:
1. Trastorno depresivo mayor con ideas delirantes incongruentes con el estado de ánimo.
2. Trastorno obsesivo compulsivo.
3. Esquizofrenia paranoide.
4. Trastorno dismórfico corporal
5. Trastorno de Somatización.
11. En relación a la disomnia señale lo correcto ?
1. Es un Trastorno en la Conducta del Sueño.
2. Es un Trastorno de inicio y de mantenimiento del Sueño.
3. Es un trastorno de la conducta y mantenimiento de sueño.
4. Ejemplos de disomnias son Pesadillas, parálisis del sueño y Enuresis.
5. Ninguna de las anteriores es correcta sobre las disomnias.
12. Mujer de 17 años, estudiante, vive con sus padres y hermanos. Angustiada llora constantemente, decaída,
desanimada, ha dejado de salir a fiestas y ya no comparte con amigos. La noche anterior ingiere cantidad no
precisada de tabletas “para los nervios” que tomaba su madre. No es primera vez que lo hace, ésta es la tercera vez
y siempre ha sido luego de un problema. Refiere estar arrepentida por lo que hizo.

Cuál es el probable trastorno de personalidad que acompaña a esta mujer que acude con criterios para depresión?

1. T. de personalidad Histriónica.
2. T. de personalidad Ciclitímica
3. T. de personalidad Obsesiva
4. T. de personalidad Esquizotípica
5. T. de personalidad Limítrofe
13. Mariano es un señor de 53 años de edad que acude a su consulta refiriendo sentir un gran malestar desde
hace ya unos años. Relaciona el malestar con una conducta que encuentra absurda pero que es incapaz de
evitar. Esta conducta que lleva a cabo siempre al llegar a su casa de vuelta del trabajo consiste en accionar
el pomo del baño del piso superior de la vivienda antes de hacer cualquier otra cosa, incluso antes de
saludar a su familia. Algunas veces ha intentado resistirse a hacerlo pero solo ha conseguido angustiarse y
demorar la conducta unos minutos. Éste comportamiento que Mariano realiza de un modo incoercible y
automático es lo que se denomina:
1. Comportamiento normal.
2. Obsesión.
3. Impulsión.
4. Compulsión.
5. Tic complejo
8 de febrero de 2018
UNIVERSIDAD CENTRAL DEL ECUADOR – FACULTAD DE CIENCIAS MÉDICAS – CÁTEDRA DE PSIQUIATRÍA.

14. Señale cual de los siguientes tratamientos NO está indicado para el tratamiento crónico del trastorno
Psicótico Breve.
1. Quetiapina
2. Olanzapina
3. Risperidona.
4. Metilfenidato.
5. Aripiprazol
.
15. Las siguientes son ejemplos de Parasomnias excepto?
1. Bruxismo nocturno
2. Erecciones dolorosas relacionadas al sueño
3. Parálisis del Sueño
4. Narcolepsia
5. Terrores Nocturnos
16. Cuál es el examen utilizado para evaluación de los trastorno de sueño.
1. Electroencefalografía durante el sueño.
2. Medición de frecuencia de movimientos oculares durante el sueño.
3. Polisomnografía.
4. Capnometría y Oximetría nocturna.
5. Biometría Hemática simple.

17. Mujer de 47 años sin antecedentes psiquiátricos que es hospitalizada para estudio de metrorragias y a los 5 días
de su ingreso es informada del diagnóstico de neoplasia uterina con metástasis. Se consulta a Psiquiatría
porque 24 horas después muestra tristeza y llanto frecuente, refiere ideas de muerte y presenta insomnio. En la
evaluación psiquiátrica no se recoge ningún antecedente psiquiátrico y la exploración detecta elevada ansiedad
y desesperanza en relación con las consecuencias de su enfermedad neoplásica. El diagnóstico más probable
es:

1. Trastorno esquizoafectivo.
2. Distimia.
3. Trastorno de ansiedad generalizada.
4. Trastorno por estrés postraumático.
5. Trastorno adaptativo.
18. Un señor de 60 años, empleado de comercio, acude a urgencias acompañado de su familia; éstos cuentan que
el día anterior, de forma bastante brusca, comenzó a decir cosas raras, a no responder a lo que le preguntaban
y a mostrar se confuso incluso respecto de su propio nombre. Efectivamente en la exploración parece no
entender lo que se le pregunta, no recuerda nada de lo que le ha pasado y no sabe ni el día, ni el lugar en el que
está. El diagnóstico sería:

1. Psicosis breve.
2. Delirium.
3. Alzheimer.
4. Esquizofrenia.
5. Trastorno de ansiedad
19. ¿En cuál de los siguientes casos en una mujer embarazada la terapia electroconvulsivante NO es un tratamiento
de primera elección?:
1. Esquizofrenia paranoide.
2. Psicosis reactiva.
3. Depresión mayor con ideación suicida.
4. Trastorno esquizotípico de la personalidad.
5. Trastorno Psicótico Breve.

8 de febrero de 2018
UNIVERSIDAD CENTRAL DEL ECUADOR – FACULTAD DE CIENCIAS MÉDICAS – CÁTEDRA DE PSIQUIATRÍA.

20. Paciente de 15 años de edad, acude a consulta ambulatoria general, acompañada de sus padres, refieren que
han encontrado en su habitación una carta de despedida al parecer con objetivos suicidas, al entrevistar a la
paciente ella niega deseos suicidas sin embargo se muestra muy renuente y negativa durante la entrevista, cuál
es su siguiente paso como médico General en esta situación?
1. Explicar a los padres que probablemente su hija necesite atención Psicológica dado que podría estar
atravesando una depresión.
2. Iniciar tratamiento antidepresivo con un antidepresivo ISRS como la Fluoxetina.
3. No dar ninguna explicación especial dado que esto es algo frecuente y la paciente está manipulando la
situación.
4. Enviar a emergencia psiquiátricas a la paciente para probable internamiento.
5. Llamar a una nueva consulta en una semana y explicar a los padres que deben estar atentos de la
paciente.
21. Cuál de estos fármacos antidepresivos no presenta como efecto colateral disminución de la líbido en los
hombres?
1. Bupropion XL.
2. Venlafaxina.
3. Fluoxetina.
4. Sertralina.
5. Mirtazapina.
22. Todos estos son fármacos útiles en el Insomnio Primario Excepto?
1. Zolpidem
2. Hidroxicina
3. Bromazepam
4. Difenhidramina
5. Metilfenidato
23. Que es el vaginismo?
1. Aumento de flora anormal en la mucosa vaginal.
2. Contracción involuntaria de las paredes musculares vaginales.
3. Infección vaginal gonocócica.
4. Resequedad de las paredes vaginales.
5. Infección vaginal indeterminada.
24. Que es el Voyeurismo?
1. Placer a través del frotamiento de órganos genitales con objetos o personas.
2. Placer con uso de correas en el acto sexual.
3. Placer en la sensación de traicionar a la pareja sentimental.
4. Placer al sentir dolor o mediante juegos de dominación durante el acto sexual.
5. Placer al ver a otras personas en situaciones eróticas.
25. Estos son criterios para trastorno de ansiedad Generalizada excepto:
1. Ansiedad y preocupación excesivas
2. Inquietud o impaciencia; fatigabilidad fácil.
3. Tensión muscular, irritabilidad
4. El Tiempo mínimo de todos los síntomas es de un mes.
5. La sintomatología no debe ser causada por drogas.
26. Cuál es el principal diagnóstico diferencial en una fuga disociativa ?
1. Convulsión.
2. Demencia de Alzheimer
3. Delirium
4. Intoxicación con drogas
5. Depresión Psicótica.
27. Qué es un heteroreceptor en la neurotransmisión ?
1. Es un receptor diferente al neurotransmisor que lo activa.
2. En un receptor capaz de activarse por diferentes Neurotransmisores.
3. Es un receptor que se sitúa en la membrana de una neurona de diferente clase en relación al ligando
que lo estimula.
4. Es un receptor atípico el cual presenta una heterogenicidad en relación a la activación de la protenia G.
5. Es un receptor que presenta dos subunidades diferentes la una con la otra.
8 de febrero de 2018
UNIVERSIDAD CENTRAL DEL ECUADOR – FACULTAD DE CIENCIAS MÉDICAS – CÁTEDRA DE PSIQUIATRÍA.

APELLIDOS Y NOMBRES……………………………………………FIRMA…………………………………………

1. Paciente sexo masculino 30 años, casado. Acude con una carpeta con documentos bajo el brazo, buen contacto,
limpio y bien vestido. Relata que hace 6 meses que está con un cuadro depresivo severo, que ha pasado por
múltiples tratamientos y diferentes médicos y no ha mejorado. Con licencia médica desde el inicio de su
enfermedad, la cual se inició tras separación conyugal (por segunda vez) porque, “ella no me comprendía”.
Asegura ser un excelente trabajador y esposo. Niega consumo de drogas y/o alcohol .Al tratar de establecer una
sintomatología precisa del momento actual, aporta datos contradictorios y poco claros. El aspecto, la presencia y
el enfoque no están de acuerdo con lo que relata (sin angustia, voz fuerte y firme, y sin compromiso emocional).
Asegura enfáticamente que no puede retomar su trabajo, razón por la que acude al Servicio de Urgencia, pidiendo
un certificado dado que el día siguiente tiene que presentarse a un proceso judicial por divorcio. Cuál es la
impresión diagnóstica más probable en este paciente?
1. T. de Personalidad.
2. T. disociativo de personalidad (Personalidad Múltiple)
3. Simulación.
4. T. por estrés postraumático.
5. T. Depresivo.
2. Profesor varón, de 70 años de edad, natural y procedente del Chota, quién desde hace 2 meses aproximadamente
y de forma reiterada insiste en un reclamo a una familia que se apellida Belaunde sobre una herencia que
supuestamente le corresponde refiere en su reclamo judicial “Mis apellidos son Vela Terry, pero para confundir a
la opinión pública y a mí persona, el apellido del ex -presidente lo han escrito con “B” y le han añadido el “unde
por lo que las herencias del presidente de la República me corresponderían”. El paciente presenta además
cambios conductuales hace aproximadamente un año que los familiares describen como que sale de la realidad
además de aislamiento y lo notan muy triste, antes del año el paciente era completamente normal ¿Señale Ud.
cuál es el diagnóstico menos probable en este caso.
1. Demencia de Alzheimer.
2. Trastorno orgánico Cerebral.
3. Esquizofrenia Paranoide.
4. Depresión con psicosis.
5. Demencia mixta vascular y senil.
3. El tiempo de tratamiento farmacológico para un paciente con diagnóstico de Esquizofrenia debe ser de:

1) 6 meses.
2) 12 meses.
3) 2 a 4 años.
4) de por vida.
5) depende de la respuesta.

4. Los tres componentes básicos para el diagnóstico de autismo son:


1. Mala Interacción social, retraso mental y falta de juego espontáneo.
2. Imposibilidad de comunicar sentimientos, retraso mental y retraso en el lenguaje.
3. Conductas restrictivas, pobre interacción social y retraso o ausencia en el desarrollo del lenguaje.
4. Déficit cognitivo, pobre interacción social y retraso en el lenguaje.
5. Afectación social/laboral; retraso en el lenguaje y que no sea causado por enfermedad médica.
5. Señale cuál de los siguientes tratamientos farmacológicos tiene mayor grado de evidencia y eficacia para el tratamiento de un paciente
con alcoholismo?
1. Olanzapina
2. Naltrexona Depot IM
3. Carbamazepina
4. Clonidina.
5. Complejo B más Tiamina.

8 de febrero de 2018
UNIVERSIDAD CENTRAL DEL ECUADOR – FACULTAD DE CIENCIAS MÉDICAS – CÁTEDRA DE PSIQUIATRÍA.

6. En qué fase del sueño se presentan ondas alfa similares al estado de vigilia ?
1. REM
2. NREM Fase 1
3. NREM Fase 2
4. NREM Fase 3
5. NREM Fase 4

7. Señalar cuáles de los siguientes síntomas de la esquizofrenia constituye un factor de mal pronóstico:
1. Pródromo breve.
2. Ausencia de síntomas depresivos.
3. Inicio tardío.
4. Ausencia de síntomas obsesivos.
5. Presencia de Síntomas negativos
8. Señalar en que trastorno de la personalidad se observa la ausencia de remordimientos y la incapacidad para
planificar el futuro:
1. Trastorno paranoide de la personalidad.
2. Trastorno límite de la personalidad.
3. Trastorno antisocial de la personalidad.
4. Trastomo de la personalidad por evitación.
5. Trastorno Dependiente de la Personalidad.
9. Hombre de 26 años, soltero, que es traído a consulta por su familia por llevar 3 meses negándose a salir se su
casa. La razón que aduce el paciente es el convencimiento de que tiene la mandíbula asimétrica y la cara
torcida. Según refiere esta situación es progresiva y cada vez se ve mas deforme cuando se observa en el
espejo. Se avergüenza de su aspecto, por lo que no quiere salir, se angustia mucho cuando ve su imagen y no
puede dejar de pensar todo el día en su deformidad. Ha consultado con varios cirujanos maxilofaciales pero
éstos le dicen que no presenta asimetría facial y lo remiten al psiquiatra. El diagnóstico del paciente es:
1. Trastorno depresivo mayor con ideas delirantes incongruentes con el estado de ánimo.
2. Trastorno obsesivo compulsivo.
3. Esquizofrenia paranoide.
4. Trastorno dismórfico corporal
5. Trastorno de Somatización.
10. En relación a la disomnia señale lo correcto ?
1. Es un Trastorno en la Conducta del Sueño.
2. Es un Trastorno de inicio y de mantenimiento del Sueño.
3. Es un trastorno de la conducta y mantenimiento de sueño.
4. Ejemplos de disomnias son Pesadillas, parálisis del sueño y Enuresis.
5. Ninguna de las anteriores es correcta sobre las disomnias.
11. Mujer de 17 años, estudiante, vive con sus padres y hermanos. Angustiada llora constantemente, decaída,
desanimada, ha dejado de salir a fiestas y ya no comparte con amigos. La noche anterior ingiere cantidad no
precisada de tabletas “para los nervios” que tomaba su madre. No es primera vez que lo hace, ésta es la tercera vez
y siempre ha sido luego de un problema. Refiere estar arrepentida por lo que hizo.

Cuál es el probable trastorno de personalidad que acompaña a esta mujer que acude con criterios para depresión?

1. T. de personalidad Histriónica.
2. T. de personalidad Ciclitímica
3. T. de personalidad Obsesiva
4. T. de personalidad Esquizotípica
5. T. de personalidad Limítrofe

8 de febrero de 2018
UNIVERSIDAD CENTRAL DEL ECUADOR – FACULTAD DE CIENCIAS MÉDICAS – CÁTEDRA DE PSIQUIATRÍA.

12. Mariano es un señor de 53 años de edad que acude a su consulta refiriendo sentir un gran malestar desde
hace ya unos años. Relaciona el malestar con una conducta que encuentra absurda pero que es incapaz de
evitar. Esta conducta que lleva a cabo siempre al llegar a su casa de vuelta del trabajo consiste en accionar
el pomo del baño del piso superior de la vivienda antes de hacer cualquier otra cosa, incluso antes de
saludar a su familia. Algunas veces ha intentado resistirse a hacerlo pero solo ha conseguido angustiarse y
demorar la conducta unos minutos. Éste comportamiento que Mariano realiza de un modo incoercible y
automático es lo que se denomina:
1. Comportamiento normal.
2. Obsesión.
3. Impulsión.
4. Compulsión.
5. Tic complejo
13. Señale cual de los siguientes tratamientos NO está indicado para el tratamiento crónico del trastorno
Psicótico Breve.
1. Quetiapina
2. Olanzapina
3. Risperidona.
4. Metilfenidato.
5. Aripiprazol
.
14. Las siguientes son ejemplos de Parasomnias excepto?
1. Bruxismo nocturno
2. Erecciones dolorosas relacionadas al sueño
3. Parálisis del Sueño
4. Narcolepsia
5. Terrores Nocturnos
15. Cuál es el examen utilizado para evaluación de los trastorno de sueño.
1. Electroencefalografía durante el sueño.
2. Medición de frecuencia de movimientos oculares durante el sueño.
3. Polisomnografía.
4. Capnometría y Oximetría nocturna.
5. Biometría Hemática simple.

16. Mujer de 47 años sin antecedentes psiquiátricos que es hospitalizada para estudio de metrorragias y a los 5 días
de su ingreso es informada del diagnóstico de neoplasia uterina con metástasis. Se consulta a Psiquiatría
porque 24 horas después muestra tristeza y llanto frecuente, refiere ideas de muerte y presenta insomnio. En la
evaluación psiquiátrica no se recoge ningún antecedente psiquiátrico y la exploración detecta elevada ansiedad
y desesperanza en relación con las consecuencias de su enfermedad neoplásica. El diagnóstico más probable
es:

1. Trastorno esquizoafectivo.
2. Distimia.
3. Trastorno de ansiedad generalizada.
4. Trastorno por estrés postraumático.
5. Trastorno adaptativo.

8 de febrero de 2018
UNIVERSIDAD CENTRAL DEL ECUADOR – FACULTAD DE CIENCIAS MÉDICAS – CÁTEDRA DE PSIQUIATRÍA.

17. Un señor de 60 años, empleado de comercio, acude a urgencias acompañado de su familia; éstos cuentan que
el día anterior, de forma bastante brusca, comenzó a decir cosas raras, a no responder a lo que le preguntaban
y a mostrar se confuso incluso respecto de su propio nombre. Efectivamente en la exploración parece no
entender lo que se le pregunta, no recuerda nada de lo que le ha pasado y no sabe ni el día, ni el lugar en el que
está. El diagnóstico sería:

1. Psicosis breve.
2. Delirium.
3. Alzheimer.
4. Esquizofrenia.
5. Trastorno de ansiedad
18. ¿En cuál de los siguientes casos en una mujer embarazada la terapia electroconvulsivante NO es un tratamiento
de primera elección?:
1. Esquizofrenia paranoide.
2. Psicosis reactiva.
3. Depresión mayor con ideación suicida.
4. Trastorno esquizotípico de la personalidad.
5. Trastorno Psicótico Breve.

19. Paciente de 15 años de edad, acude a consulta ambulatoria general, acompañada de sus padres, refieren que
han encontrado en su habitación una carta de despedida al parecer con objetivos suicidas, al entrevistar a la
paciente ella niega deseos suicidas sin embargo se muestra muy renuente y negativa durante la entrevista, cuál
es su siguiente paso como médico General en esta situación?
1. Explicar a los padres que probablemente su hija necesite atención Psicológica dado que podría estar
atravesando una depresión.
2. Iniciar tratamiento antidepresivo con un antidepresivo ISRS como la Fluoxetina.
3. No dar ninguna explicación especial dado que esto es algo frecuente y la paciente está manipulando la
situación.
4. Enviar a emergencia psiquiátricas a la paciente para probable internamiento.
5. Llamar a una nueva consulta en una semana y explicar a los padres que deben estar atentos de la
paciente.
20. Cuál de estos fármacos antidepresivos no presenta como efecto colateral disminución de la líbido en los
hombres?
1. Bupropion XL.
2. Venlafaxina.
3. Fluoxetina.
4. Sertralina.
5. Mirtazapina.
21. Todos estos son fármacos útiles en el Insomnio Primario Excepto?
1. Zolpidem
2. Hidroxicina
3. Bromazepam
4. Difenhidramina
5. Metilfenidato
22. Que es el vaginismo?
1. Aumento de flora anormal en la mucosa vaginal.
2. Contracción involuntaria de las paredes musculares vaginales.
3. Infección vaginal gonocócica.
4. Resequedad de las paredes vaginales.
5. Infección vaginal indeterminada.
23. Que es el Voyeurismo?
1. Placer a través del frotamiento de órganos genitales con objetos o personas.
2. Placer con uso de correas en el acto sexual.
3. Placer en la sensación de traicionar a la pareja sentimental.
4. Placer al sentir dolor o mediante juegos de dominación durante el acto sexual.
5. Placer al ver a otras personas en situaciones eróticas.

8 de febrero de 2018
UNIVERSIDAD CENTRAL DEL ECUADOR – FACULTAD DE CIENCIAS MÉDICAS – CÁTEDRA DE PSIQUIATRÍA.

24. Estos son criterios para trastorno de ansiedad Generalizada excepto:


1. Ansiedad y preocupación excesivas
2. Inquietud o impaciencia; fatigabilidad fácil.
3. Tensión muscular, irritabilidad
4. El Tiempo mínimo de todos los síntomas es de un mes.
5. La sintomatología no debe ser causada por drogas.
25. Cuál es el principal diagnóstico diferencial en una fuga disociativa ?
1. Convulsión.
2. Demencia de Alzheimer
3. Delirium
4. Intoxicación con drogas
5. Depresión Psicótica.
26. Qué es un heteroreceptor en la neurotransmisión ?
1. Es un receptor diferente al neurotransmisor que lo activa.
2. En un receptor capaz de activarse por diferentes Neurotransmisores.
3. Es un receptor que se sitúa en la membrana de una neurona de diferente clase en relación al ligando
que lo estimula.
4. Es un receptor atípico el cual presenta una heterogenicidad en relación a la activación de la protenia G.
5. Es un receptor que presenta dos subunidades diferentes la una con la otra.
27. Qué núcleo de la base se encuentra relacionado directamente con el abuso de sustancias?
1. Núcleo lenticular.
2. Núcleo límbico.
3. Putamen.
4. Núcleo Accumbens.
5. Globus Pálido.
6. El conjunto del Núcleo caudado
28. Cuál es el mejor tratamiento en el caso de los trastornos Somatomorfos?
1. Antidepresivos + Psicoterapia.
2. Antipsicóticos + Psicoterapia.
3. Antidepreivos + antipsicóticos + psicoterapia.
4. Estabilizadores del estado de ánimo + psicoterapia.
29. Caso clínico. Un estudiante varón de 20 años de edad acude a su consulta en su año de rural (SU RURAL LA
REALIZA EN UN CENTRO DE SALUD DE LA CIUDAD DEL COCA Y NO HAY LA ESPECIALIDAD DE
PSIQUIATRÍA EN EL CENTRO DE SALUD Y TAMPOCO EN EL HOSPITAL DEL COCA), previamente sano, se
ha estado comportando de forma cada vez más extraña. A veces parece enfadado, comenta a sus amigos que
está siendo seguido por la policía y por servicios secretos, a veces ríe solo sin motivo. Últimamente se preocupa
menos por su aseo personal y su rendimiento académico ha decaido. Cuando fue a consulta estaba inquieto y
asustado. Decía que escuchaba voces que comentaban sus acciones. Además piensa que le están robando sus
pensamientos y que en la televisión hacen referencia a él, estos síntomas inician según sus padres hace dos
meses y han ido empeorando. Ud luego de realizar la HCL completa, frente al paciente y los familiares cual sería
su siguiente paso?
1. Por tratarse de una probable psicosis debería ser transferido al Hospital Del Coca a pesar de no contar
con psiquiatras en dicho Hospital.
2. Por ser una emergencia psiquiátrica evaluaría la probabilidad de autoagresividad y heteroagresividad así
como los deseo de escapar del paciente e instauraría un tratamiento con haloperidol IM probablemente 15
mg.
3. Me contactaría con el jefe del Centro de Salud para saber como proceder a transferir a este paciente a un
Hospital Psiquiátrico.
4. Realizaría un hemograma general incluyendo hormonas tiroideas, además de realización de una
tomografía de urgencia en caso que el centro de salud conste con estas posibilidades.
5. Todas son posibilidades correctas.
6. Solo la 2 y la 3 son correctas.
30. Tratamiento farmacológico de primera línea para el trastorno obsesivo compulsivo.
1. Sertralina 50 mg PO AM diario.
2. Paroxetina CR 25mg 2 Tab HS diaria.
3. Fluoxetina 20mg PO AM diaria
4. Terapia Psicológica ambulatoria.
5. Todas son terapias de primera línea.
6. Solo 1 y 3 son terapias de primera línea.
8 de febrero de 2018
UNIVERSIDAD CENTRAL DEL ECUADOR – FACULTAD DE CIENCIAS MÉDICAS – CÁTEDRA DE PSIQUIATRÍA.

31. Muchacha de 19 años estudiante, sin antecedentes personales somáticos ni psiquiátricos relevantes que es
llevada a urgencias por su familia por ingesta masiva de pastillas hace una hora. Siendo la paciente encontrada
por su madre casualmente al volver a su casa antes de lo que tenía previsto. Paciente refiere presentar desde
unas 4 semanas atrás un cuadro de tristeza, anhedonia, desesperanza, sentimientos de culpa y deseos de
muerte, inhibición psicomotriz y enlentecimiento del pensamiento, dificultad de concentración y rendimiento en
los estudios, aislamiento social, hipersomnia diurna y empeoramiento matutino de los síntomas. Así mismo la
paciente refería tener la sensación de que sus vecinos la espiaban, hablaban de ella cuando salía y se reían
de ella lo que había incrementado su angustia. No sabía el motivo por el que la gente se fijaba en ella pero
estaba convencida de que no eran imaginaciones suyas. Señale el mejor tratamiento en urgencias.

1. Amitriptilina intravenosa y Haloperidol Intravenoso.


2. Carbón activado por sonda nasogástrica.
3. Fluoxetina 60mg a altas dosis por vía oral todas las mañanas.
4. Sertralina 50mg en la mañana y Olanzapina 5mg en la noche.
5. Bupropion XL 150mg en las noches y Quetiapina 300 a 600 mg todas las mañanas.
32. La presencia de síntomas orgánicos no causados voluntariamente sin explicación médica se explicaría mejor por
cuál de las siguientes enfermedades psiquiátricas?
1. Trastorno facticio.
2. Simulación.
3. Trastorno facticio por poderes.
4. Trastorno somatomorfo
5. Todas son correctas
33. Los tres componentes básicos para el diagnóstico de autismo son:
1. Mala Interacción social, retraso mental y falta de juego espontáneo.
2. Imposibilidad de comunicar sentimientos, retraso mental y retraso en el lenguaje.
3. Conductas restrictivas, pobre interacción social y retraso o ausencia en el desarrollo del lenguaje.
4. Déficit cognitivo, pobre interacción social y retraso en el lenguaje.
5. Afectación social/laboral; retraso en el lenguaje y que no sea causado por enfermedad médica.
34. Qué fármaco podría causar un brote maniacal en un tratamiento de un paciente con depresión crónica tratada por más de 40 años con
diferentes antidepresivos?.
1. Bupropion (Wellbutrim)
2. Todos podrían.
3. Sertralina.
4. Venlafaxina.
5. Ninguno podría.
35. Cuál de los siguientes fármacos no se usa en el tratamiento del trastorno Bipolar?
1. Olanzapina.
2. Acd. Valpróico.
3. Fluoxetina
4. Carbonato de Litio
5. Todos estos fármacos se pueden usar.

8 de febrero de 2018
NOMBRE COMPLETO:____________________________________

FIRMA:___________________ FECHA________________________

1. Qué núcleo de la base se encuentra relacionado directamente con el abuso de sustancias?


1. Núcleo lenticular.
2. Núcleo límbico.
3. Putamen.
4. Núcleo Accumbens.
5. Globus Pálido.
6. El conjunto del Núcleo caudado
2. Cuál es el mejor tratamiento en el caso de los trastornos Somatomorfos?
1. Antidepresivos + Psicoterapia.
2. Antipsicóticos + Psicoterapia.
3. Antidepreivos + antipsicóticos + psicoterapia.
4. Estabilizadores del estado de ánimo + psicoterapia.
3. Caso clínico. Un estudiante varón de 20 años de edad acude a su consulta en su año de rural (SU RURAL LA
REALIZA EN UN CENTRO DE SALUD DE LA CIUDAD DEL COCA Y NO HAY LA ESPECIALIDAD DE
PSIQUIATRÍA EN EL CENTRO DE SALUD Y TAMPOCO EN EL HOSPITAL DEL COCA), previamente sano,
se ha estado comportando de forma cada vez más extraña. A veces parece enfadado, comenta a sus amigos
que está siendo seguido por la policía y por servicios secretos, a veces ríe solo sin motivo. Últimamente se
preocupa menos por su aseo personal y su rendimiento académico ha recaÍdo. Cuando fue a consulta estaba
inquieto y asustado. Decía que escuchaba voces que comentaban sus acciones. Además piensa que le están
robando sus pensamientos y que en la televisión hacen referencia a él, estos síntomas inician según sus
padres hace dos meses y han ido empeorando. Ud luego de realizar la HCL completa, frente al paciente y los
familiares cual sería su siguiente paso?
1. Por tratarse de una probable psicosis debería ser transferido al Hospital Del Coca a pesar de no contar
con psiquiatras en dicho Hospital.
2. Por ser una emergencia psiquiátrica evaluaría la probabilidad de autoagresividad y heteroagresividad
así como los deseo de escapar del paciente e instauraría un tratamiento con haloperidol IM
probablemente 10mg.
3. Me contactaría con el jefe del Centro de Salud para saber como proceder a transferir a este paciente
a un Hospital Psiquiátrico.
4. Todas son posibilidades correctas.
5. Solo la 2 y la 3 son correctas
4. Paciente sexo masculino 30 años, casado. Acude con una carpeta con documentos bajo el brazo, buen
contacto, limpio y bien vestido. Relata que hace 6 meses que está con un cuadro depresivo severo, que ha
pasado por múltiples tratamientos y diferentes médicos y no ha mejorado. Con licencia médica desde el inicio
de su enfermedad, la cual se inició tras separación conyugal (por segunda vez) porque, “ella no me
comprendía”. Asegura ser un excelente trabajador y esposo. Niega consumo de drogas y/o alcohol .Al tratar
de establecer una sintomatología precisa del momento actual, aporta datos contradictorios y poco claros. El
aspecto, la presencia y el enfoque no están de acuerdo con lo que relata (sin angustia, voz fuerte y firme, y
sin compromiso emocional). Asegura enfáticamente que no puede retomar su trabajo, razón por la que acude
al Servicio de Urgencia, pidiendo un certificado dado que el día siguiente tiene que presentarse a un proceso
judicial por divorcio. Cuál es la impresión diagnóstica más probable en este paciente?
1. T. de Personalidad.
2. T. disociativo de personalidad (Personalidad Múltiple)
3. Simulación.
4. T. por estrés posttraumático.
5. T. Depresivo.

5. Tratamiento farmacológico de primera línea para el trastorno obsesivo compulsivo.


1. Sertralina 50 mg PO AM diario.
2. Paroxetina CR 25mg 2 Tab HS diaria.
3. Fluoxetina 20mg PO AM diaria
4. Terapia Psicológica ambulatoria.
5. Todas son terapias de primera línea.
6. Solo 1 y 3 son terapias de primera línea.
6. Muchacha de 19 años estudiante, sin antecedentes personales somáticos ni psiquiátricos relevantes que
es llevada a urgencias hospitalarias por su familia por ingesta masiva de pastillas hace una hora. Siendo
la paciente encontrada por su madre casualmente al volver a su casa antes de lo que tenía previsto.
Paciente refiere presentar desde unas 4 semanas atrás un cuadro de tristeza, anhedonia, desesperanza,
sentimientos de culpa y deseos de muerte, inhibición psicomotriz y enlentecimiento del pensamiento,
dificultad de concentración y rendimiento en los estudios, aislamiento social, hipersomnia diurna y
empeoramiento matutino de los síntomas. Así mismo la paciente refería tener la sensación de que sus
vecinos la espiaban, hablaban de ella cuando salía y se reían de ella lo que había incrementado su
angustia. No sabía el motivo por el que la gente se fijaba en ella pero estaba convencida de que no eran
imaginaciones suyas. Señale el mejor tratamiento farmacológico de mantenimiento de su probable
enfermedad psiquiátrica

1. Amitriptilina intravenosa y Haloperidol Intravenoso.


2. Carbón activado por sonda nasogástrica.
3. Fluoxetina 60mg a altas dosis por vía oral todas las mañanas.
4. Sertralina 50mg en la mañana y Olanzapina 5mg en la noche.
5. Bupropion XL 150mg en las noches y Quetiapina 300 a 600 mg todas las mañanas.

7. La presencia de síntomas orgánicos no causados voluntariamente sin explicación médica se explicaría mejor
por cuál de las siguientes enfermedades psiquiátricas?
1. Trastorno facticio.
2. Simulación.
3. Trastorno disociativo.
4. Trastorno somatomorfo
5. Todas son correctas
8. Profesor varón, de 70 años de edad, natural y procedente del Chota, quién desde hace 2 meses
aproximadamente y de forma reiterada insiste en un reclamo a una familia que se apellida Belaunde sobre
una herencia que supuestamente le corresponde refiere en su reclamo judicial “Mis apellidos son Vela Terry,
pero para confundir a la opinión pública y a mí persona, el apellido del ex -presidente lo han escrito con “B” y
le han añadido el “unde por lo que las herencias del presidente de la República me corresponderían”. El
paciente presenta además cambios conductuales hace aproximadamente un año que los familiares describen
como que sale de la realidad, antes del año el paciente era completamente normal ¿Señale Ud. cuál es el
diagnóstico menos probable en este caso.
1. Demencia de Alzheimer.
2. Trastorno orgánico Cerebral.
3. Esquizofrenia Paranoide.
4. Depresión con psicosis.
5. Demencia mixta vascular y senil.
9. Una mujer de 23 años, sin historia de abuso de alcohol, refiere episodios matinales de temblor, ansiedad,
taquicardia y sudoración. ¿Cuál de los siguientes exámenes tiene mayor probabilidad de estar alterado?

1. Amonemia
2. Glicemia
3. Potasio sérico
4. T3, T4, TSH
5. Pesquisa de drogas en orina

10. El tiempo de tratamiento farmacológico para un paciente con diagnóstico de Esquizofrenia debe ser de:

1) 6 meses.
2) 12 meses.
3) 2 a 4 años.
4) de por vida.
5) depende de la respuesta.

11. Los tres componentes básicos para el diagnóstico de autismo son:


1. Mala Interacción social, retraso mental y falta de juego espontáneo.
2. Imposibilidad de comunicar sentimientos, retraso mental y retraso en el lenguaje.
3. Conductas restrictivas, pobre interacción social y retraso o ausencia en el desarrollo del lenguaje.
4. Déficit cognitivo, pobre interacción social y retraso en el lenguaje.
5. Afectación social/laboral; retraso en el lenguaje y que no sea causado por enfermedad médica.
6.
12. Señale cuál de los siguientes tratamientos farmacológicos tiene mayor grado de evidencia y eficacia para
el tratamiento de un paciente con alcoholismo?
1. Olanzapina
2. Naltrexona Depot IM
3. Carbamazepina
4. Antabus.
5. Complejo B más Tiamina.
13. Señalar cual es el trastorno donde el paciente tiende a acudir al médico frecuentemente por el simple hecho
de buscar atención médica que incluso llega al hecho de enfermarse voluntariamente.
1. Trastorno de personalidad narcisista.
2. Trastornos Somatomorfo.
3. Trastorno Dismórfico Corporal.
4. Trastorno Hipocondriaco.
5. Trastorno facticio.
14. Qué fármaco podría causar un brote maniacal en un tratamiento de un paciente con depresión crónica de 40
años de evolución?.
1. Bupropion (Wellbutrim)
2. Todos podrían.
3. Sertralina.
4. Venlafaxina.
5. Ninguno podría.
15. Cuál de los siguientes fármacos no se usa en el tratamiento del trastorno Bipolar?
1. Olanzapina.
2. Acd. Valpróico.
3. Fluoxetina
4. Carbonato de Litio
5. Todos estos fármacos se pueden usar.
16. Que es la dependencia o adicción a las drogas? Necesidad Psicológica de ingerir una droga
compulsivamente.
1. Necesidad Física de ingerir una droga compulsivamente
2. Necesidad Psicológica y física de usar una droga compulsivamente
3. Necesidad de aumento progresivo de dosis y ansiedad intensa al momento de suspender el consumo.
4. Necesidad de buscar nuevas drogas y presencia de alteraciones laborales, ambientales por su
consumo.
17. Cuál es la escalera en el tratamiento de dolor según la OMS en el tratamiento de dolor somático(ejemplo)?
1. Solo Paracetamol, luego paracetamol con diclofenaco, luego inhibidores de la COX 2, luego Fentanilo.
2. Solo Paracetamol, luego paracetamol con diclofenaco, luego paracetamol con diclofenaco con codeina,
luego paracetamol y morfina.
3. Solo Paracetamol, luego solo ketorolaco, luego solo tramadolo, luego solo morfina.
4. Solo paracetamol, luego un AINE, Luego opioides potentes como la Codeina o Tramadol
5. Ninguna es correcta.
18. Paciente de 25 años que acude a emergencias por palpitaciones, sudoración, palidez, temblor fino en manos,
primer episodio en su vida. Cual diagnóstico psiquiátrico es el menos probable en el caso de este paciente.
1. Abuso de Drogas.
2. Ataque de Pánico.
3. Fobia Social.
4. Psicosis Aguda.
5. Intento de Suicidio.
6. Desrealización Disociativos
19. Cuál de estos no forma parte de la posible presentación de un Trastorno Bipolar tipo uno?
1. Estado Depresivo.
2. Estado Psicótico.
3. Estado Mixto.
4. Sin patología.
5. Estado hipomaniacal.
20. Mujer de 17 años, estudiante, vive con sus padres y hermanos. Angustiada llora constantemente, decaída,
desanimada, ha dejado de salir a fiestas y ya no comparte con amigos. La noche anterior ingiere cantidad no
precisada de tabletas “para los nervios” que tomaba su madre. No es primera vez que lo hace, ésta es la tercera
vez y siempre ha sido luego de un problema. Refiere estar arrepentida por lo que hizo. Cuál es el probable trastorno
de personalidad que acompaña a esta mujer que acude con criterios para depresión?
1. T. de personalidad Histriónica.
2. T. de personalidad Ciclotímica
3. T. de personalidad Obsesiva
4. T. de personalidad Esquizotípica
5. T. de personalidad Limítrofe

También podría gustarte